SSC CGL Tier-I 11 June 2019 Shift-I Previous Year Paper

SSC CGL

(11 June 2019 Shift-I) 

Reasoning 

Instructions For the following questions answer them individually 

Q.1 Select the word pair in which the two words are related in the same way as the two words in the following word-pair. Brass : Copper 

A Bronze : Gold 

B Chromium : Silver 

C Solder : Tin 

D Nickle : Iron 

Answer: C 

 

Q.2 Priyank is Akshay’s brother. Sonia is Saksham’s sister. Akshay is Sonia’s son. Howis Priyank related to Sonia? 

A Brother 

B Father 

C Nephew 

D Son 

Answer: D 

 

Q.3 Select the figure that will come next in the following figure series. 

Answer: D 

 

Q.4 Find the missing number from the below options. 

A 34 

B 32 

C 21 

D 12 

Answer: C 

 

Q.5 Select the correct mirror image of the given figure when the mirror is placed to the right of the figure. 

Answer: B 

 

Q.6 If AU = 21 and EGG = 245, then how will you code BAKE? 

A 75 

B 110 

C 155 

D 19 

Answer: B 

 

Q.7 Select the figure in which the given figure is embedded. 

Answer: D 

 

Q.8 Three of the following four letter-clusters are alike in a certain way and one is different. Pick the odd one out. 

A FGE 

B YZX 

C NOR 

D KLJ 

Answer: C 

 

Q.9 A square paper is folded and cut as shown below. How will it appear when unfolded ? 

Answer: D 

 

Q.10 Arrange the following words in a logical and meaningful order.

1) Write 2) Publish 3) Author 4) Reader 5) Sale 

A 3, 1, 2, 5, 4 

B 4, 3, 1, 2, 5 

C 3, 2, 1, 5, 4 

D 1, 3, 2, 5, 4 

Answer: A 

 

Q.11 Two statements are given, followed by two conclusions numbered I and II. Assuming the statements to be true, even if they seem to beat variance with commonly known facts, decide which of the conclusions logically follow(s) from the statements. 

Statements: 

All teachers are painters. 

Some painters are rich. 

Conclusions: 

I. All painters are teachers. 

II. Some rich are not painters. 

A Only conclusion II follows 

B Neither conclusion I nor II follows 

C Only conclusion I follows 

D Either conclusion I or II follows 

Answer: B 

 

Q.12 Select the terms that will replace the ? in the following series. 

Z, X, V, ?, R, P, ? 

A S, N 

B T, O 

C T, N 

D S, O 

Answer: C 

 

Q.13 Three of the following four number-pairs are alike in a certain way and one is different. Pick the odd one out. 

A 17-37 

B 49-82 

C 65-101 

D 26-50 

Answer: B 

 

Q.14 Choose the Venn diagram from the given options which best represents the relationship amongst the following classes: Wheat, Mustard, Cabbage 

Answer: A 

 

Q.15 In a code language, NEEDLE is written as MFDEKF. How will HAMMER be written in the same language? 

A IBLNDS 

B GBLNFT 

C GBLNDS 

D GALNDS 

Answer: C 

 

Q.16 How many triangles are there in the following figure ? 

A 30 

B 22 

C 16 

D 32 

Answer: A 

 

Q.17 ‘Cheerful’ is related to ‘Sad’ in the same way as ‘Generous’ is related to ……….. 

A Intelligent 

B Selfish 

C Kind 

D Gloomy 

Answer: B 

 

Q.18 Three different positions of a dice are shown below. Which number appears on the face opposite the number 6 ? 

A 4 

B 5 

C 2 

D 1 

Answer: B 

 

Q.19 Find the missing number from the below options. 

A 90 

B 80 

C 100 

D 72 

Answer: A

 

Q.20 At a party, the number of girls is half the number of boys. After an hour, five boys leave the party and three girls join the party. How many people were present at the party an hour before? 

A 22 

B 8 

C 16 

D 24 

Answer: D 

 

Q.21 Select the term that will come next in the following series. 

57, 62, 31, 36, 18, ? 

A 23 

B 36 

C 34 

D 19 

Answer: A 

 

Q.22 Three of the following four words are alike in a certain way and one is different. Pick the odd word out. 

A Thorium 

B Uranium 

C Sodium 

D Radium 

Answer: C 

 

Q.23 Select the option that is related to the third number in the same way as the second number is related to the first number. 2139 : 3246 :: 4262 : ……… 

A 2371 

B 2471 

C 1461 

D 2483 

Answer: B 

 

Q.24 Select the term that will come next in the following series. OBE, PDH, QFK, RHN, ? 

A RJQ 

B SJP 

C SJQ 

D SKQ 

Answer: C 

 

Q.25 Which two signs should be interchanged in the following equation to make it correct? 

24 ÷ 8 − 5 × 5 + 3 = 13 

A × and

B ÷ and × 

C × and − 

D ÷ and

Answer: A 

 

General knowledge 

Instructions For the following questions answer them individually 

Q.26 In India, 24th January is observed as which one of the following days? 

A National Farmer’s Day 

B National Girl Child Day 

C National Agricultural Day 

D National Day of Elderly 

Answer: B 


Q.27 The Sepoy Mutiny in India started from ______. 

A Meerut 

B Champaran 

C Rajkot 

D Bareilly 

Answer: A 

 

Q.28 Fertile riverine alluvial soil is best suited for producing ______. 

A rice 

B Corn 

C tea 

D cotton 

Answer: A 

 

Q.29 Jorwe culture’ was a Chalcolithic archaeological site located in the present day Indian state of ______. 

A Bihar 

B Assam 

C Gujarat 

D Maharashtra 

Answer: D 

 

Q.30 Vijay Hazare was a famous Indian player associated with the sport of ______. 

A Cricket 

B Boxing 

C Football 

D Rifle Shooting 

Answer: A 

 

Q.31 Name the national bird of Bhutan. 

A Magpai 

B Parrot 

C Raven 

D Peacock 

Answer: C 

 

Q.32 Which scientist discovered the ‘Penicillin’? 

A Robert Koch 

B Ernst Chain 

C Alexander Fleming 

D Louis Pasteur 

Answer: C

 

Q.33 The energy derived from the heat of Earth is called ______. 

A Tidal Energy 

B Geothermal Energy 

C Biogas 

D Solar Energy 

Answer: B 

 

Q.34 The term ‘cherry picking’ is used in which sport? 

A Cricket 

B Table Tennis 

C Basketball 

D Swimming 

Answer: C 

 

Q.35 Which economist gave the theory of Opportunity Cost? 

A Adam Smith 

B Gottfried Haberler 

C Milton Friedman 

D John Keynes 

Answer: B 

 

Q.36 Which of the following authors received the Jnanpith Award in 2018? 

A Arundhati Roy 

B Vikram Seth 

C Amitav Ghosh 

D Kiran Desai 

Answer: C 

 

Q.37 Which of the following acids is present in ant bites? 

A Perchloric acid 

B Formic acid 

C Nitric acid 

D Malic acid 

Answer: B 

 

Q.38 Marfati’ songs are traditional folk songs of ______. 

A Pakistan 

B Bangladesh 

C Nepal 

D Afghanistan 

Answer: B 

 

Q.39 The summer solstice 2019 in the Northern Hemisphere will occur on ______. 

A 21st June 

B 20th June 

C 24th June 

D 26th June 

Answer: A 

 

Q.40 The demand for a commodity or service which is a consequence of the demand for something else is called ______. 

A Direct Demand 

B Income Demand 

C Derived Demand 

D Composite Demand 

Answer: C 

 

Q.41 Who among the following wrote the book ‘A History of the Sikhs’? 

A Amrita Pritam 

B Bhai Vir Singh 

C Khushwant Singh 

D Gurbachan Singh Talib 

Answer: C 

 

Q.42 Which of the following creatures is oviparous? 

A Squirrel 

B Rabbit 

C Mouse 

D Frog 

Answer: D 

 

Q.43 Which of the following stars is nearest to the sun ? 

A Deneb 

B Sirius 

C Betelgeuse 

D Proxima Centauri 

Answer: D 

 

Q.44 Which article of the Constitution of India talks about the provisions for impeachment of the President of India? 

A Article 54 

B Article 61 

C Article 51 

D Article 63 

Answer: B 

 

Q.45 Identify the foreigner who has been conferred the second highest civilian award in India, Padma Vibhushan for ‘exceptional and distinguished service’ in 2019. 

A John Chambers 

B Kader Khan 

C Pravin Gordhan 

D Ismail Omar Guelleh 

Answer: D 

 

Q.46 Who among the following was named as the Chief Executive Officer of International Cricket Council (ICC)? 

A Manu Sawhney 

B Anurag Thakur 

C Saurav Ganguly 

D N G Khaitan 

Answer: A 

 

Q.47 In which year was the East India Company incorporated for the exploitation of trade with East and Southeast Asia and India? 

A 1600 

B 1596 

C 1612 

D 1605 

Answer: A 

 

Q.48 Which of the following is the smallest bird in the world? 

A Bee Hummingbird 

B Finch 

C Diamond Firetail 

D Robin 

Answer: A 

 

Q.49 What is the term of the elected representatives of a Gram Panchayat? 

A 2 years 

B 4 years 

C 5 years 

D 3 years 

Answer: C 

 

Q.50 Who was the Indian Army Chief at the time of Bangladesh Liberation War? 

A K M Cariappa 

B Roy Bucher 

C Sam Manekshaw 

D Rob Lockhart 

Answer: C 

 

Quantitative Aptitude 

Instructions For the following questions answer them individually 

Q.51 The difference between the compound interest and simple interest on ₹ x at 8.5% per annum for 2 years is ₹ 28.90. The value of x is: 

A 3800 

B 3500 

C 4000 

D 4500 

Answer: C 

 

Q.52 If a + b + c = 8 and ab + bc + ca = 12, then a3 + b3 + c3 − 3abc is equal to: 

A 144 

B 400 

C 192 

D 224 

Answer: D 

 

Q.53 If a : b = 5 : 3, then (8a — 5b) : (8a + 5b) is equal to: 

A 2:5 

B 3:11 

C 5:11 

D 3:13 

Answer: C 

 

Q.54 If tan4θ = cot(2θ + 30 ), then θ is equal to: 

A 20

B 25 

C 15

D 10

Answer: D 

 

Q.55 The following table indicates the number of students studying in three disciplines in five colleges: 

What percentage of total students are studying in the commerce stream in all five colleges together? 

A 28% 

B 30% 

C 32% 

D 33% 

Answer: B 

 

Q.56 Two articles are sold for ₹ 10,005 each. On one,the seller gains 15% and onthe other, he loses 13%. What is his overall gain or loss percent, correct to two decimal places? 

A 1.42% gain 

B 0.94% loss 

C 0.94% gain 

D 1.42% loss 

Answer: B 

 

Q.57 △ABC ∼ △QRP and PQ = 6 cm, QR = 8 cm and PR = 10 cm.If ar( △ABC) : ar( △PQR) = 1 : 4, then AB is equal to: 

A 2 cm 

B 3 cm 

C 4 cm 

D 5 cm 

Answer: C 

 

Q.58 A train without stop page travels with an average speed of 65 km/h and with stoppage,it travels with an average speed of 52 km/h. For how many minutes does the train stop on an average per hour? 

A 14 

B 13 

C 12 

D 15 

Answer: C 

 

Q.59 Chords AB and CD of a circle, when produced, meet at a point P outside the circle. If AB = 6 cm, CD = 3 cm and PD = 5 cm, then PB is equal to: 

A 4 cm 

B 6.25 cm 

C 5 cm 

D 6 cm 

Answer: A 

 

Q.60 A is 40% more efficient than B and C is 20% less efficient than B. Working together, they can finish a task in 15 days. In how many days, will B alone complete 75% of the task? 

A 48 

B 44 

C 36 

D 32 

Answer: C

 

Q.61 The price of sugar has increased by 22 % . A person wants to increase his expenditure by 12% only. By what percent should he decrease his consumption, nearest to one decimal place? 

A 10% 

B 8.2% 

C 7.8% 

D 8.6% 

Answer: B 

 

Q.62 If √x + 1/√x = √6, then x2+1/x2is equal to: 

A 36 

B 16 

C 62 

D 14 

Answer: D 

 

Q.63 An article is sold for ₹ 657.90 after successive discounts of 15 % and 10%. What is the marked price of the article? 

A ₹880 

B ₹920 

C ₹860 

D ₹900 

Answer: C 

 

Q.64 The value of sec2 28 − cot2 62 + sin2 60 + cosec2 30is equal to: 

A 19/4

B 3 

C 7/2

D 23/4

Answer: D 

 

Q.65 The following table indicates the number of students studying in three disciplines in five colleges: 

What is the ratio of the total number of students studying in the science stream to that of studying in commerce stream in all five colleges taken together? 

A 16:15 

B 19:15 

C 16:19 

D 14:15 

Answer: A 

 

Q.66 The radii of two circular faces of the frustum of a cone of height 21 cm are 3 cm and 2 cm respectively. What is the volume of the frustum of the cone in cm3 (π = 22/7 )? 

A 154 

B 418 

C 345 

D 286 

Answer: B 

 

Q.67 In △ABC, AD is a median and is a point on AD such that AP : PD =3 : 4. Then ar( △APB) : ar( △ABC) is equal to: 

A 3 : 14 

B 3 : 7 

C 3 : 4 

D 2 : 7 

Answer: A 

 

Q.68 In a circle with centre O, AB is the diameter and CD is a chord such that ABCD is a trapezium. If ∠BAC =25 , then ∠CAD is equal to: 

A 40

B 25

C 65

D 45

Answer: A 

 

Q.69 If cotθ = 3/4, then sinθ + cosθ − tanθ is equal to: 

A ⅟15

B −⅟20 

C ⅟20

D ²/₁₅

Answer: A 

 

Q.70 If a + b = 5 and ab = 3, then (a3 + b3 )is equal to: 

A 80 

B 70 

C 75 

D 65 

Answer: A 

 

Q.71 The value of: 28 + (5.2 ÷ 1.3 × 2) − 6 × 3 ÷ 8 + 2 

A 4.55 

B 6.45 

C 10.55 

D 8.45 

Answer: C 

 

Q.72 What is the least value of x such that 517×324 is divisible by 12? 

A 3 

B 2 

C 1 

D 0 

Answer: B 

 

Instructions 

The following table indicates the number of students studying in three disciplines in five colleges: 

Q.73 What percentage of students in college B is studying in the science stream, (correct to one decimal place)? 

A 29.4% 

B 29.2% 

C 29.6% 

D 29.8% 

Answer: B 

 

Q.74 If a pie-chart is drawn representing the number of students in all five colleges, what is the central angle (correct to the nearest whole number) of the sector representing the students of college B? 

A 86∘ 

B 82∘ 

C 80∘ 

D 84∘ 

Answer: A 

 

Instructions For the following questions answer them individually 

Q.75 The average marks of 45 students was found to be 66. If the marks of two students were incorrectly entered as 28 and 64 instead of 82 and 46respectively, then what is the correct average? 

A 67.2 

B 66.8 

C 66.4 

D 66.6 

Answer: B 

 

English 

Instructions For the following questions answer them individually 

Q.76 Select the correct active form of the given sentence. We will all be greatly benefitted by this scheme. 

A This scheme is going to greatly benefit us all. 

B This scheme would greatly benefit we all. 

C This scheme has greatly benefitted us all. 

D This scheme will greatly benefit us all. 

Answer: D 

 

Q.77 Select the most appropriate meaning of the underlined idiom in the given sentence. Due to an increased number of layoffs in the industry, the sword of Damocles is always hanging over the employees. 

A strict rules and regulations 

B an ill omen of death 

C threat of physical harm 

D a constant threat 

Answer: D 

 

Q.78 Given below are four jumbled sentences. Select the option that gives their correct order. 

A. The brain is active too during sleep, sending messages for the heart to beat regularly. 

B. However, the body utilizes the sleeping time effectively. 

C. We spend about one third of our time sleeping. 

D. It produces energy and releases hormones for repair and growth during the night. 

A ABDC 

B CABD 

C CBDA 

D BADC 

Answer: C 

 

Q.79 Select the word which means the same as the group of words given. A fictitious name used by an author 

A alibi 

B pesudonym 

C anonymous 

D homonym 

Answer: B 

 

Q.80 In the sentence identify the segment which contains the grammatical error. Ten kilometres are a long distance to cover on foot for a child. 

A to cover on foot 

B a long distance 

C Ten kilometres are 

D for a child 

Answer: C 

 

Q.81 Select the most appropriate option to substitute the underlined segment in the given sentence. If no substitution is required, select No improvement. Knowing the particular things that motivate each person help you add power to their motivation. 

A helps you add power 

B No improvement 

C help you adding power 

D help you to add power 

Answer: A 

 

Q.82 Select the most appropriate synonym of the given word. PERPLEX 

A deceive 

B complex 

C bewilder 

D surprise 

Answer: C 

 

Instructions: In the following passage some words have been deleted. Fill in the blanks with the help of the alternatives given. Select the most appropriate option for each blank. 

Comprehension: Leaving his car with the valet, he (1)______ the hotel and joined the large crowd milling (2)______ He found a badge waiting for him (3)______ the receptionist’s table and took his (4)______ in the rear of the hall. When the place was (5)______ the moderator welcomed the crowd. 

Q.83 Select the most appropriate option to fill in blank No.1. 

A entered 

B admitted 

C accessed 

D invaded 

Answer: A 

 

Q.84 Select the most appropriate option to fill in blank No.2. 

A about 

B aside 

C into 

D out 

Answer: A 

 

Q.85 Select the most appropriate option to fill in blank No.3. 

A over 

B by 

C in 

D at 

Answer: D 

 

Q.86 Select the most appropriate option to fill in blank No.4. 

A lounge 

B couch 

C stand 

D seat 

Answer: D 

 

Q.87 Select the most appropriate option to fill in blank No.5. 

A empty 

B filled 

C completed 

D stuffed 

Answer: B 

 

Instructions For the following questions answer them individually 

Q.88 In the sentence identify the segment which contains the grammatical error. There isn’t many rice left in the house so we must replenish our stock soon. 

A There isn’t many rice 

B left in the house 

C our stock soon 

D so we must replenish 

Answer: A 

 

Q.89 Select the correctly spelt word. 

A tuetion 

B tution 

C tuttion 

D tuition 

Answer: D 

 

Q.90 Select the most appropriate antonym of the given word. MEAGRE 

A plentiful 

B scanty 

C inadequate 

D premium 

Answer: A 

 

Q.91 Select the most appropriate meaning of the underlined idiom in the given sentence. True friends stay by our side through thick and thin. 

A under all circumstances 

B in difficult times 

C in happy moments 

D in different weathers 

Answer: A 

 

Q.92 Select the most appropriate synonym of the given word. DEFER 

A dictate 

B dread 

C delay 

D despair 

Answer: C 

 

Q.93 Select the correct passive form of the given sentence. The sailors had never encountered such a rough sea. 

A Such a rough sea had never been encountered by the sailors. 

B Such a rough sea is never encountered by the sailors. 

C Such a rough sea has never been encountered by the sailors. 

D Such a rough sea was never encountered by the sailors. 

Answer: A 

 

Q.94 Select the most appropriate option to fill in the blank. She got a lucrative job of a translator because she was ______ in French. 

A deficient 

B sufficient 

C proficient 

D efficient 

Answer: C 

 

Q.95 Select the most appropriate option to fill in the blank. The committee reached ______ decision regarding the appointment of the chairman. 

A a compatible 

B a unanimous 

C an agreeable 

D an exemplary 

Answer: B 

 

Q.96 Select the word which means the same as the group of words given. One who is a great lover of books 

A hemophile 

B pedophile 

C bibliophile 

D xenophile 

Answer: C 

 

Q.97 Select the most appropriate option to substitute the underlined segment in the given sentence. If no substitution is required, select No improvement. Your coming home to dinner on time should be a rule rather the exception. 

A should be a rule rather being the exception 

B No improvement 

C shall be a rule rather than a exception 

D should be the rule rather than the exception 

Answer: D 

 

Q.98 Given below are four jumbled sentences. Select the option that gives their correct order. 

A. But they faced grave danger if they tried to criticize these decisions. 

B. The nationalists now began to openly criticize the policies of the British. 

C. The freedom movement changed this situation. 

D. Under colonial rule, the people had lived in fear of the British government and did not agree with many of the decisions that they took. 

A CBAD 

B DBAC 

C DACB 

D BADC 

Answer: C 

 

Q.99 Select the correctly spelt word. 

A previllage 

B peivailege 

C privilege 

D previledge 

Answer: C 

 

Q.100 Select the most appropriate antonym of the given word. ARROGANCE 

A superiority 

B sweetness 

C vanity 

D humility 

Answer: D 

SSC CGL Tier-I 11 June 2019 Shift-II Previous Year Paper

SSC CGL 

(11 June 2019 Shift-II) 

Reasoning 

Instructions For the following questions answer them individually 

Q.1 Introducing Rukmani, Vijay said, “She is my wife’s only brother’s son’s mother.” How is Rukmani’s husband related to Vijay? 

A Brother-in-law 

B Father-in-law 

C Son 

D Father 

Answer: A 

 

Q.2 Two different positions of a dice are shown below. Which number appears on the face opposite the number ‘1’? 

A 2 

B 3 

C 5 

D 4 

Answer: B 

 

Q.3 Select the term that will come next in the following series. 2, 7, 17, 32, 52, ? 

A 77 

B 126 

C 102 

D 134 

Answer: A 

 

Q.4 Select the correct mirror image of the given figure when the mirror is placed to the right of the figure. 

Answer: A 

 

Q.5 Find the missing number from the below options. 

A 208 

B 232 

C 116 

D 142 

Answer: A 

 

Q.6 ‘Stag’ is related to ‘Doe’ in the same wayas ‘Bull’ is related to ………… 

A Vixen 

B Ox 

C Mare 

D Cow 

Answer: D 

 

Q.7 A square paper is folded and cut as shown below. How will it appear when unfolded ? 

Answer: D 

 

Q.8 Select the figure that will come next in the following figure series. 

Answer: B 

 

Q.9 Find the missing number from the below options. 

A 5 

B 9 

C 6 

D 7 

Answer: D 

 

Q.10 Select the option that is related to the third word in the same way as the second word is related to the first word. Potato : Starch :: Stevia : ? 

A Salt 

B Butter 

C Sugar 

D Milk 

Answer: C 

 

Q.11 Two statements are given, followed by two conclusions numbered I and II. Assuming the statements to be true, even if they seem to be at variance with commonly known facts, decide which of the conclusions logically follow(s) from the statements. 

Statements: 

All humans are parents. 

All parents are teachers. 

Conclusions: 

I. Some teachers are parents. 

II. Some teachers are humans. 

A Either conclusion I or II follows 

B Only conclusion II follows 

C Both the conclusions follow 

D Only conclusion I follows 

Answer: C 

 

Q.12 Vishal is three times as old as Saksham. After 8 years, he will be two times as old as Saksham. After further 8 years, what will be Vishal’s age? 

A 48 years 

B 24 years 

C 40 years 

D 32 years 

Answer: C 

 

Q.13 Select the terms that will replace ? in the following series.

K, M, O, ?, S, U, ?, Y 

A Q, X 

B Q, W 

C P, W 

D R, W 

Answer: B 

 

Q.14 Select the figure in which the given figure is embedded. 

Answer: D 

 

Q.15 Three of the following four letter-clusters are alike in a certain way and one is different. Pick the odd one out. 

A DWKO 

B HSYB 

C QJUF 

D GTXC 

Answer: A 

 

Q.16 Which two signs should be interchanged to make the given equation correct? 

12 × 8 ÷ 36 + 3 − 6 = 102 

A + and − 

B ÷ and

C × and

D ÷ and × 

Answer: B 

 

Q.17 Arrange the following words in a logical and meaningful order. 1. Brass 2. Gold 3. Silver 4. Platinum 5. Iron 

A 2, 1, 5, 3, 4 

B 5, 4, 3, 2, 1 

C 3, 1, 5, 2, 4 

D 5, 1, 3, 2, 4 

Answer: D 

 

Q.18 If N = 28 and ORE = 76, then how will you code PALE? 

A 72 

B 68 

C 19 

D 76 

Answer: B 

 

Q.19 In a code language, PAINTER is written as SCLPWGU. How will WRITER be written in the same language? 

A ZTLVGV 

B ZTLVHU 

C ZTLVHT 

D YTLVHT 

Answer: C

 

Q.20 Select the letter cluster that will come next in the following series. 

GZI, HYJ, IXK, JWL, ? 

A KVM 

B KUM 

C JVN 

D LVM 

Answer: A 

 

Q.21 Select the option that is related to the third number in the same way as the second number is related to the first number. 9 : 86 : 13 : …… 

A 175 

B 135 

C 174 

D 124 

Answer: C 

 

Q.22 Three of the following four number-pairs are alike in a certain way and one is different. Pick the odd one out. 

A 13-234 

B 7-126 

C 11-198 

D 9-164 

Answer: D 

 

Q.23 Choose the Venn diagram from the given options which best represents the relationship amongst the following classes: Cup, Table, Crockery 

Answer: B 

 

Q.24 Three of the following four words are alike in a certain way and one is different. Pick the odd word out. 

A Earth 

B Jupiter 

C Mars 

D Metis 

Answer: D 

 

Q.25 How many triangles are there in the following figure ? 

A 34 

B 30 

C 28 

D 32 

Answer: D 

 

General knowledge 

Instructions For the following questions answer them individually 

Q.26 In January, _____ was named World No.1 boxer by International Boxing Association (AIBA). 

A Mary Kom 

B Laishram Sarita Devi 

C Simranjit Kaur 

D Lovlina Borgohain 

Answer: A 

 

Q.27 In which year did Vasco De Gama land in Calicut (Kozhikode)? 

A 1498 

B 1458 

C 1472 

D 1442 

Answer: A 

 

Q.28 Who among the following was the first president of the Republic of China? 

A Yuan Shikai 

B Hu Jintao 

C Li Xiannian 

D Yang Sangkun 

Answer: A 

 

Q.29 Who among the following is/was the longest serving prime minister of Bangladesh? 

A Shah Azizur Rahman 

B Khaleda Zia 

C Sheikh Hasina 

D Sheikh Mujibur Rahman 

Answer: C 

 

Q.30 India of My Dreams’ is a compilation of the writings and speeches of ______. 

A Jawaharlal Nehru 

B Dr Rajendra Prasad 

C Mahatma Gandhi 

D Sardar Vallabhbhai Patel 

Answer: C 

 

Q.31 Manipur, Meghalaya and Tripura became states under ______. 

A North Eastern Retention (Reconstruction) Act, 1971 

B North Eastern Republic of India Act, 1972 

C North Eastern Region New State Act, 1972 

D North Eastern Areas (Reorganisation) Act, 1971 

Answer: D 

 

Q.32 Name the woman of Indian origin who was appointed Chief Executive Officer of PepsiCo in the year 2006. 

A Kiran Majumdar Shaw 

B Kalpana Morparia 

C Indra Nooyi 

D Chanda Kochhar 

Answer: C

 

Q.33 As per Mankiw’s Principles of Economics, the standard of living of a country depends on the country’s ______. 

A nominal wages 

B government policy 

C ability to produce goods and services 

D average wages 

Answer: C 

 

Q.34 Identify the unit of measuring intensity of sound. 

A Decibels 

B Knots 

C Candela 

D Ampere 

Answer: A 

 

Q.35 In which year did the Patharughat Peasant Uprising against the tax policies of British take place in Assam? 

A 1873 

B 1862 

C 1885 

D 1894 

Answer: D 

 

Q.36 Daringbadi hill station is located in the Indian state of ______. 

A Maharashtra 

B Kerala 

C West Bengal 

D Odisha 

Answer: D 

 

Q.37 Who among the following was the first chief minister of Kerala? 

A E M S Namboodiripad 

B Pattom A Thanu Pillai 

C R Shankar 

D C Achutha Menon 

Answer: A 

 

Q.38 Which Indian state launched the PRANAM Commission to protect parents of government employees? 

A Assam 

B Haryana 

C Maharashtra 

D Punjab 

Answer: A 

 

Q.39 Who discovered the first vaccine for smallpox? 

A Louis Pasteur 

B John Hunter 

C Alexander Fleming 

D Edward Jenner 

Answer: D 

 

Q.40 In January 2019, _____ became the first Indian state to implement 10% reservation for the economically weaker sections. 

A Tripura 

B Madhya Pradesh 

C Gujarat 

D Odisha 

Answer: C 

 

Q.41 When the output is equal to zero, the variable cost is ______. 

A constant 

B zero 

C minimum 

D maximum 

Answer: B 

 

Q.42 The ‘Diphu Pass’ which is the tri-junction between India, Myanmar and China is on this Border Line? 

A Durand Line 

B Radcliffe Line 

C McMahon Line 

D Palk Strait 

Answer: C 

 

Q.43 Which instrument is used to measure the intensity of light produced by an unknown source in terms of a standard source? 

A Dynamometer 

B Calipers 

C Photometer 

D Ammeter 

Answer: C 

 

Q.44 Which of the following is the third highest water fall in India? 

A Thalaiyar Falls 

B Courtallam Falls 

C Suruli Falls 

D Agaya Gangai 

Answer: A 

 

Q.45 Kandyan Dance’ is a typical dance form practiced in ______. 

A Nepal 

B Sri Lanka 

C India 

D Bhutan 

Answer: B 

 

Q.46 Which Indian archer won the gold medal in the women’s recurve event at the 2018 Hyundai Archery World Cup? 

A Bombayla Devi Laishram 

B Chekrovolu Swuro 

C Deepika Kumari 

D Dola Banerjee 

Answer: C 

 

Q.47 The Indian badminton player who won the Canadian Open Women’s Doubles title in 2015 along with Ashwini Ponnappa was _____. 

A P V Sindhu 

B P C Tulasi 

C Saina Nehwal 

D Jwala Gutta 

Answer: D 

 

Q.48 Which of the following is the chemical name of baking soda? 

A Sulphate 

B Sodium carbonate 

C Sodium hydrogen carbonate 

D Calcium hydroxide 

Answer: C 

 

Q.49 Kazi Nazrul Islam is the national poet of ______. 

A Indonesia 

B Pakistan 

C Afghanistan 

D Bangladesh 

Answer: D 

 

Q.50 According to _____, pressure is equal to the force divided by the area on which it acts. 

A Hooke’s Law 

B Newton’s Law 

C Pascal’s Law 

D Stefan-Boltzmann Law 

Answer: C 

 

Quantitative Aptitude 

Instructions For the following questions answer them individually 

Q.51 An article is sold for ₹1,680 after two successive discounts of 20% and 16%. What is the marked price of the article? 

A ₹2,400 

B ₹2,500 

C ₹2,200 

D ₹2,300 

Answer: B 

 

Q.52 If cosecθ = 13/12, then sinθ + cosθ − tanθ is equal to: 

A 139/65 

B 91/65

C −71/65 

D 71/65

Answer: C 

 

Q.53 △ABC ∼ △PRQ and PQ = 4 cm, QR = 7 cm and PR = 8 cm. If ar( △ABC) : ar( △PQR) = 1 : 4, then AC is equal to: 

A 2 cm 

B 3.7 cm 

C 4 cm 

D 1 cm 

Answer: A 

 

Q.54 The table below indicates the percentage of students and the ratio of boys and girls in the various streams of a college. (Total students = 2600) 

In which stream, is the difference in the percentage of boys and girls minimum? 

A ME 

B EC 

C CS 

D IT 

Answer: B 

 

Q.55 If a : b = 2 : 3 and c : b = 5 : 6, then a : b : c is equal to: 

A 10 : 15 : 18 

B 6 : 9 : 16 

C 4 : 6 : 5 

D 6 : 9 : 12 

Answer: C 

 

Q.56 In a circle with centre O, AB is the diameter and CD is a chord such that ABCD is a trapezium. If ∠BAC = 18, then ∠CAD is equal to: 

A 18

B 72

C 36

D 54

Answer: D 

 

Q.57 If the six digit number 4x573y is divisible by 72 then the value of (x + y) is: 

A 4 

B 6 

C 8 

D 9 

Answer: C 

 

Q.58 In △ABC, AD is a median and is a point on AD such that AP : PD = 3 : 4. Then ar(△BPD) : ar(△ABC) is equal to: 

A 2 : 7 

B 4 : 7 

C 2 : 5 

D 1 : 3 

Answer: A 

 

Q.59 A train without stoppage travels with an average speed of 72 km/h and with stoppage,it travels with an average speed of 60 km/h. For how many minutes does the train stop on an average per hour? 

A 10 

B 6 

C 12 

D 8 

Answer: A 

 

Q.60 The value of cot262 − sec228 + cosec230 + tan260 is equal to: 

A 16/3

B 8 

C 10/3

D 6 

Answer: D 

 

Q.61 In a class of 40 students, 45% are girls and the remaining are boys.If the average of the girls’ marks is 54 and that of the boys is 46, what is the average of the whole class? 

A 49.6 

B 49.7 

C 49.8 

D 49.5 

Answer: A 

 

Q.62 The difference between the compound interest and simple interest on ₹x at 7.5% per annum for 2 years is ₹45. What is the value of x? 

A 9,000 

B 7,000 

C 10,000 

D 8,000 

Answer: D 

 

Q.63 Two articles are sold for ₹5,104 each. On one, the seller gains 16% and on the other, he loses 12%. What is his overall gain percent, nearest to two decimal places? 

A 0.10% 

B 0.08% 

C 0.12% 

D 0.14% 

Answer: B 

 

Q.64 Two chords AB and CD of a circle when produced, meet at a point P outside the circle. If AB = 6 cm, PB = 5 cm, PD = 4 cm, then CD is equal to: 

A 7.5 cm 

B 8.25 cm 

C 9.75 cm 

D 7.75 cm 

Answer: C 

 

Q.65 The radii of the two circular faces of the frustum of a cone are 5 cm and 4 cm.If the height of the frustum is 21 cm, what is its volume in cm3 ? (π = 22/7) 

A 902 

B 638 

C 1056 

D 1342 

Answer: D 

 

Q.66 If (a+b) = 6 and ab =16/3 , then (a3 + b3) is equal to: 

A 120 

B 190 

C 220 

D 150 

Answer: A 

 

Q.67 A is 40% more efficient than B and C is 20% less efficient than B. Working together, they can complete a task in 20 hours. In how many hours will A alone complete 35% of that task? 

A 14 

B 15 

C 13 

D 16 

Answer: D 

 

Q.68 The table below indicates the percentage of students and the ratio of boys and girls in the various streams of a college. (Total students = 2600) 

If the data about the number of girls enrolled in the various streams is represented by a pie-chart, what is the central angle of the sector representing the number of girls in the ME stream, to the nearest whole degree? 

A 72∘ 

B 70∘ 

C 74∘ 

D 68∘ 

Answer: A 

 

Q.69 If a + b + c = 6 and ab + bc + ca = 4, then a3 + b3 + c3 − 3abc is equal to: 

A 154 

B 144 

C 148 

D 160 

Answer: B 

 

Q.70 The price of sugar is increased by 17%. A person wants to increase his expenditure by 8% only. By what percent should he decrease his consumption, nearest to one decimal place? 

A 7.9% 

B 8.1% 

C 8.3% 

D 7.7% 

Answer: D 

 

Q.71 If √x 1/√x= √6 , then x2 + 1/x2 is equal to: 

A 54 

B 62 

C 66 

D 40 

Answer: B 

 

Q.72 The value of: 7.2 + (8.4 ÷ 0.12 × 0.2) − 5 × 3 ÷ 0.054 + 3 

A -275.8 

B -75.8 

C 21.2 

D -175.8 

Answer: A 

 

Q.73 If sinθ = cos(50 ∘ + θ), then θ is equal to: 

A 20

B 35

C 25

D 30

Answer: A 

 

Instructions: The table below indicates the percentage of students and the ratio of boys and girls in the various streams of a college. (Total students = 2600) 

Q.74 What is the ratio of boys and girls in the college? 

A 1 : 1 

B 6 : 7 

C 5 : 6 

D 7 : 8 

Answer: A 

 

Q.75 What is the ratio of students studying in CS and IT? 

A 12 : 13 

B 6 : 7 

C 9 : 11 

D 11 : 13 

Answer: B 

 

English 

Instructions For the following questions answer them individually 

Q.76 Select the most appropriate option to substitute the underlined segment in the given sentence. If no substitution is required, select No improvement. The big farmers with deepest tube wells still have water, but many others face a water crisis. 

A in deep tubewells 

B No improvement 

C through deepest tubewells 

D with deeper tubewells 

Answer: D 

 

Q.77 Select the word which means the same as the group of words given. A place for storing guns and military equipment 

A aviary 

B archive 

C arsenal 

D apiary 

Answer: C 

 

Q.78 Select the correctly spelt word. 

A afliction 

B exhibition 

C tution 

D voilation 

Answer: B 

 

Q.79 Select the most appropriate antonym of the given word. CRUCIAL 

A imperative 

B pivotal 

C critical 

D trivial 

Answer: D 

 

Q.80 Given below are four jumbled sentences. Select the option that gives their correct order. 

A. But the rest of the students were rushing past her for a break, like she didn’t exist. 

B. She slowly pushed open the library door and took a seat at one of the tables in the corner. C. And today, Jessica really felt like maybe she didn’t exist. 

D. When the bell rang for lunch, Jessica took her lunch box and started moving towards the library. 

A BDCA 

B DACB 

C DBCA 

D CADB 

Answer: B 

 

Q.81 Select the most appropriate option to fill in the blank. The ship sailed smoothly owing to ______ winds. 

A tempestuous 

B favourable 

C turbulent 

D boisterous 

Answer: B 

 

Q.82 Given below are four jumbled sentences. Select the option that gives their correct order. 

A. Then, they spread as the unmined coal starts burning with oxygen drawn from pores and mine shafts. 

B. An entire township- Jharia is to be relocated because of the uncontrollable underground fires. 

C. These fires start mostly from burning trash close to coal pits. 

D. This is a grave threat as poisonous fumes of carbon monoxide rise up from underground fires. 

A CDAB 

B BDAC 

C BCAD 

D DCAB 

Answer: C 

 

Q.83 Select the correctly spelt word. 

A desperete 

B desparate 

C disparat 

D desperate 

Answer: D 

 

Q.84 Select the most appropriate synonym of the given word. TACITURN 

A reticent 

B phlegmatic 

C placid 

D talkative 

Answer: A 

 

Q.85 In the sentence identify the segment which contains the grammatical error. The length of a male swallow’s tail reveal his attractiveness for a female swallow. 

A a female swallow 

B a male swallow’s tail reveal 

C The length of 

D his attractiveness for 

Answer: B 

 

Q.86 Select the most appropriate synonym of the given word. MAMMOTH 

A miniscule 

B magnificent 

C gigantic 

D perilous 

Answer: C 

 

Q.87 Select the most appropriate meaning of the underlined idiom in the given sentence. The boss is going to blow his top when he discovers the blatant mistake in the balance sheet. 

A attack fiercely 

B be very embarrassed 

C be very angry 

D dismiss from job 

Answer: C 

 

Q.88 Select the correct active form of the given sentence. Heavy taxes have been imposed on luxury items by the government. 

A The government is imposing heavy taxes on luxury items. 

B The government has imposed heavy taxes on luxury items. 

C The government imposed heavy taxes on luxury items. 

D The government had imposed heavy taxes on luxury items. 

Answer: B 

 

Q.89 In the sentence identify the segment which contains the grammatical error. My father dissuaded me to try for a job as he wanted me to pursue higher studies. 

A My father dissuaded me 

B to pursue higher studies 

C to try for a job 

D as he wanted me 

Answer: C 

 

Q.90 Select the most appropriate meaning of the underlined idiom in the given sentence. A mountaineer has to walk the tight rope as a small slip can prove to be fatal. 

A be very cautious 

B be very nervous 

C be an expert 

D be well trained 

Answer: A 

 

Q.91 Select the most appropriate option to substitute the underlined segment in the given sentence. If no substitution is required, select No improvement. Some agitating miners allegation that there is no emergency measures inside the mines. 

A No improvement 

B miners alleged that there were 

C miners’ allege that there is 

D miner’s allegation that there are 

Answer: B 

 

Q.92 Select the word which means the same as the group of words given. A place where fruit trees are grown 

A orchard 

B garden 

C plantation 

D farm 

Answer: A 

 

Q.93 Select the correct passive form of the given sentence. She never trusted anyone. 

A No one was ever trusted by her 

B Anyone is not trusted by her. 

C Her trust was never for anyone. 

D She was never trusted by anyone. 

Answer: A 

 

Instructions: In the following passage some words have been deleted. Fill in the blanks with the help of the alternatives given. Select the most appropriate option for each blank. 

Comprehension: Ironically, the dams that were constructed to (1)______ floods have triggered floods due to sedimentation in the reservoir. The (2)______ of water from dams during heavy rains aggravated- the flood situation (3)______ Maharashtra and Gujarat in 2006. The floods (4)______ not only life and property but also caused soil erosion. Sedimentation (5)______ the plains of silt, a natural fertilizer.

Q.94 Select the most appropriate option to fill in blank No.1 

A dominate 

B resist 

C reduce 

D control 

Answer: D

 

Q.95 Select the most appropriate option to fill in blank No.2 

A restraint 

B liberation 

C collection 

D release 

Answer: D 

 

Q.96 Select the most appropriate option to fill in blank No.3 

A along 

B among 

C through 

D In 

Answer: D 

 

Q.97 Select the most appropriate option to fill in blank No.4 

A devastated 

B plundered 

C smashed 

D devoured 

Answer: A 

 

Q.98 Select the most appropriate option to fill in blank No.5 

A deprived 

B destroyed 

C disabled 

D distributed 

Answer: A 

 

Instructions For the following questions answer them individually 

Q.99 Select the most appropriate antonym of the given word. COVETOUS 

A benevolent 

B mercenary 

C avaricious 

D acquisitive 

Answer: A 

 

Q.100 Select the most appropriate option to fill in the blank. A ______ speech is certainly more effective than one which is verbose. 

A laconic 

B sullen 

C lengthy 

D surly 

Answer: A 

SSC CGL Tier-I 12 June 2019 Shift-III Previous Year Paper

SSC CGL

(12 June 2019 Shift-III) 

Reasoning 

Instructions For the following questions answer them individually 

Q.1 Select the figure in which the given figure is embedded. (Rotation is not allowed). 

Answer: A 

 

Q.2 In the given Venn diagram, the rectangle represents ‘women’, the pentagon represents ‘entrepreneurs’ and the triangle represents ‘mothers’. The numbers given in the diagram represent the number of persons in that particular category. 

How many women are entrepreneurs but NOT mothers? 

A 21 

B 74 

C 50 

D 46 

Answer: B 

 

Q.3 Select the option that depicts how the given transparent sheet of paper would appear if it is folded at the dotted line. 

Answer: D 

 

Q.4 Two statements are given, followed by three conclusions numbered I, II and III. Assuming the statements to be true, even if they seem to be at variance with commonly known facts, decide which of the conclusions logically follow(s) from the statements.

Statements: 

All owls are parrots. 

Some parrots are crows. 

Conclusions: 

I. No owl is a crow. 

II. All parrots are owls. 

III. Some owls are crows. 

A All the conclusions, I, II and III, follow 

B Only conclusions II and III follow. 

C Either conclusion I or III follows 

D Either conclusion I or II follows 

Answer: C 

 

Q.5 Select the option that is related to the third number in the same way as the second number is related to the first number. 3 : 30 :: 4 : ? 

A 64 

B 70 

C 68 

D 66 

Answer: C 

 

Q.6 Which two signs should be interchanged to make the following equation correct? 

18 + 24 − 6 × 6 ÷ 3 = 39

A ÷ and +

B + and −

C  × and +

D ÷ and − 

Answer: D 

 

Q.7 Three of the following four words are alike in a certain way and one is different. Pick the odd word out. 

A Frustration 

B Anaemia 

C Diabetes 

D Hypertension 

Answer: A 

 

Q.8 ‘Similarity’ is related to ‘Difference’ in the same way as ‘Transparent’ is related to ‘ ……. ’ 

A Water 

B Clear 

C Turbid 

D Lake 

Answer: C 

 

Q.9 Whichletter-cluster will replace the question mark (?) in the following series? 

SWA, OAW, KES, ?, CMK 

A GHO 

B FIP 

C GIO 

D FIO 

Answer: C 

 

Q.10 Arrange the following words in a logical and meaningful order. 

1. Ploughing up the field 2. Bread making 3. Production of wheat 4. Wheat grinding 5. Sowing the seeds 6. Dough making 

A 2, 5, 2, 4, 1, 6 

B 4, 5, 2, 6, 1, 3 

C 1, 5, 3, 6, 4, 2 

D 1, 5, 3, 4, 6, 2 

Answer: D 

 

Q.11 Which letter will replace the question mark (?) in the following series? 

D, J, ?, S, V, X 

A P 

B Q 

C M 

D O 

Answer: D 

 

Q.12 Three different positions of the same dice are shown. Which symbol will be on the face opposite to the one having ‘*’ ?

A @ 

B ! 

C $ 

D + 

Answer: D 

 

Q.13 How many triangles are there in the following figure. 

A 19 

B 21 

C 23 

D 20 

Answer: B 

 

Q.14 Which number will replace the question mark (?) in the following series? 

102, 110, 126, 150, 182, ? 

A 220 

B 246 

C 222 

D 214 

Answer: C 

 

Q.15 Three of the following four letter-clusters are alike in a certain way and one is different. Pick the odd one out. 

A ZXUQL 

B NLHDC 

C PNKGB 

D SQNIE 

Answer: B 

 

Q.16 ‘A + B’ means ‘A is the wife of B’. ‘A – B’ means ‘A is the husband of B’. ‘A ×B’ means ‘A is the son of B’. ‘A ÷ B’ means ‘A is the mother of B’. If T + Q × P – U ÷ R ÷ S + V, then how is R related to Q ? 

A Mother 

B Daughter 

C Maternal grandmother 

D Sister 

Answer: D 

 

Q.17 Three of the following four number-pairs are alike in a certain way and one is different. Pick the number- pair that is different from the rest. 

A 3 : 27 

B 4 : 65 

C 2 : 9 

D 1 : 2 

Answer: A 

 

Q.18 Select the missing number from the given options. 

A 36 

B 5 

C 8 

D 27 

Answer: B 

 

Q.19 In a code language, DISTRIBUTION is written as SIDIRTTUBNOI. How will ABBREVIATING be written in that language? 

A BBAVERTAIGNI 

B VERBBAGNITAI 

C BABRVEAITTGN 

D BBAVERIATGNI 

Answer: A

 

Q.20 Select the figure that will come next in the following figure series. 

Answer: D 

 

Q.21 The sum of the current ages of Vinayak and his father is 50 years. 5 years from now, Vinayak’s age will be one-fifth of his father’s age. Whatis Vinayak’s current age? 

A 5 years 

B 8 years 

C 15 years 

D 10 years 

Answer: A 

 

Q.22 Select the missing number from the given options.

A 43 

B 41 

C 40 

D 42 

Answer: C 

 

Q.23 Select the correct mirror image of the given figure when a vertical mirror is placed on the right of the figure. 

Answer: B 

 

Q.24 Select the word-pair in which the two words are related in the same way as the two words in the following word-pair. Accident : Injury 

A Farmer : Hard-work 

B Infection : Disease 

C Income : Corruption 

D Police : Fine 

Answer: B 

 

Q.25 In a code language, CLIMAX is written as 312913124. How will RESIGN be written as in that language? 

A 185199714 

B 185199713 

C 195209714 

D 185198614 

Answer: A 

 

General knowledge 

Instructions For the following questions answer them individually 

Q.26 British East India Company defeated the Portuguese in the ______. 

A Battle of Buxar 

B Battle of Plassey 

C Battle of Suvali 

D Battle of Chamkaur 

Answer: C 

 

Q.27 In China, 2019 (which starts on 5 February and ends on 24 January) is the year of the ________. 

A Rat 

B Dog 

C Pig 

D Ox 

Answer: C 

 

Q.28 Which team won the Indian Premier League 2018? 

A Royal Challengers Bangalore 

B Delhi Daredevils 

C Rajasthan Royals 

D Chennai Super Kings 

Answer: D 

 

Q.29 In March 2019, the Directorate General of Civil Aviation (DGCA) of India grounded which aircraft due to technical fault? 

A Boeing 787 Max 14 

B Boeing 737 Max 8 

C Boeing 777 Max 12 

D Boing 747 Max 9 

Answer: B 

 

Q.30 _____ gases trap heat in the atmosphere which makes the Earth warmer, causing global warming. 

A Greenhouse 

B Nobel 

C Elemental 

D Compound 

Answer: A 

 

Q.31 Both, haemoglobin and myoglobin require _________ for formation. 

A Iron 

B Vitamin B12 

C Calcium 

D Vitamin A 

Answer: A 

 

Q.32 Who among the following was the first Chinese to receive the Nobel Prize for literature ? 

A Mo Yan 

B Liu Cixin 

C Gao Xingjian 

D Lu Xun 

Answer: A

 

Q.33 Which of the following is the longest river in Pakistan that originates from Lake Manasarovar? 

A Chenab 

B Indus 

C Sutlej 

D Kabul 

Answer: B 

 

Q.34 Which country won the South Asian Football Federation (SAFF) Women’s Championship in 2019? 

A Bhutan 

B India 

C Pakistan 

D Nepal 

Answer: B 

 

Q.35 Opium wars were fought between the British and ______. 

A Myanmar (Burma) 

B China 

C Afghanistan 

D Bhutan 

Answer: B 

 

Q.36 Which Indian author wrote the book ‘Two Lives’? 

A Amish Tripathi 

B Chetan Bhagat 

C Vikram Seth 

D Amitava Ghosh 

Answer: C 

 

Q.37 A _____ is not constrained by prior work. 

A strategic alliance 

B franchise 

C joint venture 

D greenfield project 

Answer: D 

 

Q.38 Which human body part can be called the ‘chemical factory’ of our body? 

A Liver 

B Stomach 

C Kidneys 

D Lungs 

Answer: A 

 

Q.39 Identify the national sport of Sri Lanka. 

A Cricket 

B Volleyball 

C Basketball 

D Swimming 

Answer: B 

 

Q.40 Rule _____ (of the Rules of Procedure and Conduct of Business in Lok Sabha) does NOT involve a formal motion before the Parliament House, hence no voting can take place after discussion on matters under this rule. 

A 149 

B 186 

C 193 

D 158 

Answer: C 

 

Q.41 Nitrous Oxide’ is the chemical name of ______. 

A Tear Gas 

B Fire Extinguisher 

C Laughing Gas 

D Mosquito Repellent 

Answer: C 

 

Q.42 Which country hosted the 2019 Sultan Azlan Shah Cup hockey tournament? 

A Malaysia 

B Singapore 

C India 

D China 

Answer: A 

 

Q.43 A scientific study of rocks that deals with their composition, texture and structure; their occurrence and distribution; and their origin in relation to physicochemical conditions is called ______. 

A Petrology 

B Geomorphology 

C Geology 

D Lithology 

Answer: A 

 

Q.44 A market place in which a final good or service is bought and sold is called ______. 

A Commodity Market 

B Factor Market 

C Product Market 

D Equity Market 

Answer: C 

 

Q.45 Which one of the following is the deepest gorge in the world? 

A Kali Gandaki Gorge 

B Garganta del Cares 

C Vikos Gorge 

D Tiger Leaping Gorge 

Answer: A 

 

Q.46 Bihar Diwas is annually celebrated on _________ to commemorate the day Bihar was carved out from the Bengal Presidency. 

A 13th March 

B 27th March 

C 22nd March 

D 17th March 

Answer: C 

 

Q.47 Sir Thomas Roe visited the court of Mughal ruler _________ as the ambassador of the King of England. 

A Shah Jahan 

B Humayun 

C Akbar 

D Jahangir 

Answer: D 

 

Q.48 Tipu Sultan and British East India Company signed the Treaty of Mangalore in the year ______. 

A 1792 

B 1764 

C 1799 

D 1784 

Answer: D 

 

Q.49 How many provinces are there in Sri Lanka? 

A 6 

B 11 

C 9 

D 8 

Answer: C 

 

Q.50 In India, which of the following articles of the Constitution of India provides for the formation of a new state? 

A Article 2 

B Article 3 

C Article 9 

D Article 1 

Answer: B 

 

Quantitative Aptitude 

Instructions For the following questions answer them individually 

Q.51 What is the value of x so that the seven digit number 5656×52 is divisible by 72? 

A 8 

B 4 

C 5 

D 7 

Answer: D 

 

Q.52 This table shows the percentage of students passing out of five different colleges over three years.It is given that from each college, 200 students appeared every year. 

What is the ratio of the number of students passing to those failing from college E in the year 2015? 

A 17 : 7 

B 9 : 5 

C 18 : 7 

D 4 : 3 

Answer: C 

 

Q.53 Two articles are sold for ₹975 each. On one,the seller gains 30% andon the other, he loses 25%. What is the overall gain or loss percentage, correct to one decimal place? 

A 4.9% loss 

B 5.3% gain 

C 4.9% gain 

D 5.1% loss 

Answer: A 

 

Q.54 An article is sold for ₹612 after successive discounts of 25% and x%. If the marked price of the article is ₹960, what is the value of x? 

A 10 

B 15 

C 12 

D 14 

Answer: B 

 

Q.55 In a circle with centre O, an are ABC subtends an angle of 140 at the centre of the circle. The chord AB is produced to point P. Then ∠ CBP is equal to: 

A 70 

B 40 

C 50 

D 80 

Answer: A 

 

Q.56 If a + b + c = 7 and ab + bc + ca = 1, then a3 + b3 + c3 − 3abc is equal to: 

A 412 

B 322 

C 325 

D 422 

Answer: B 

 

Q.57 In a circle of radius 13 cm, a chord is at a distance of 12 cm from the centre of the circle. What is the length of the chord? 

A 5 cm 

B 10 cm 

C 7 cm 

D 9 cm 

Answer: B 

 

Q.58 This table shows the percentage of students passing out of five different colleges over three years.It is given that from each college, 200 students appeared every year. 

If the number of passed out students of all five colleges is represented by a pie chart, what is the central angle (to nearest whole number)of the sector representing the passed out students of college C? 

A 69 

B 77 

C 79 

D 67 

Answer: C 

 

Q.59 In △ABC,P is a point on BC such the BP : PC = 4 : 3 and is the midpoint of BP. Then ar( △ABQ): ar( △ACB)is equal to: 

A 1 : 5 

B 3 : 7 

C 4 : 7 

D 2 : 7 

Answer: D 

 

Q.60 The price of sugar has increased by 24%. A person wants to increase his expenditure by 18% only. By approximately what percent should he decrease his consumption? 

A 5.3% 

B 5.1% 

C 4.6% 

D 4.8% 

Answer: D

 

Q.61 The value 108 ÷ 36 × 4 + 2.5 × 4 ÷ 0.5 − 10 of: 

A 20 

B 22 

C 16 

D 18 

Answer: B 

 

Q.62 △ABC ∼ △EDF and ar( △ABC) : ar( △DEF) = 4 : 9. If AB = 6 cm, BC = 8 cm and AC = 10 cm,then DF is equal to: 

A 9 cm 

B 12 cm 

C 15 cm 

D 18 cm 

Answer: B 

 

Q.63 This table shows the percentage of students passing out of five different colleges over three years. It is given that from each college, 200 students appeared every year. 

What is the approximate percentage increase in the number of students passing out of college B in the year 2017 as compared to the previous year? 

A 13% 

B 13.2% 

C 12.8% 

D 13.4% 

Answer: B 

 

Q.64 If 12sinθ = 5cosθ, then sinθ + cosθ − cotθ is equal to: 

A −71/65 

B 139/156 

C 116/156 

D −16/65

Answer: A 

 

Q.65 A sphere of radius 4 cm is melted and recast into smaller spheres of radii 2 cm each. How many such spheres can be made? 

A 8 

B 32 

C 4 

D 16 

Answer: A 

 

Q.66 If √x − 1/√x= 3√2 , then x2 + 1/x2is equal to: 

A 324 

B 326 

C 402 

D 398 

Answer: D 

 

Q.67 Walking at ⅗ of his usual speed, a person reaches his office 20 minutes later than the usual time. His usual time in minutes is: 

A 40 

B 25 

C 30 

D 20 

Answer: C 

 

Q.68 In a class of 40 students, 60% are girls. The average of the girls’ marks is 72 and that of the boys is 54. What are the average marks of the whole class? 

A 65 

B 64.8 

C 65.4 

D 65.2 

Answer: B 

 

Q.69 sec2 29 − cot2 61 + sin2 60 + cosec2 30 is equal to: 

A 23/4

B 11/4

C 19/4

D 15/4

Answer: A 

 

Q.70 The efficiencies of A, B and C are in the ratio 2 : 5 : 3. Working together, they can complete a task in 9 days. In how many days will C alone complete 40% of that task? 

A 15 

B 14 

C 12 

D 16 

Answer: C 

 

Q.71 If a : b = 2 : 5, a : b = 3 : 4, then a : b : c is equal to: 

A 2 : 5 : 4 

B 6 : 15 : 20 

C 2 : 5 : 3 

D 8 : 20 : 15 

Answer: D 

 

Q.72 If a – b = 5 and ab = 2, then (a3 − b3) is equal to: 

A 155 

B 125 

C 145 

D 95 

Answer: A 

 

Q.73 The difference between the compound interest and simple interest on ₹x at 7% per annum for 2 years is ₹24.50. What is the value ofx? 

A 5,000 

B 4,800 

C 6,000 

D 5,400 

Answer: A 

 

Q.74 This table shows the percentage of students passing out of five different colleges over three years. It is given that from each college, 200 students appeared every year. 

In which college the average percentage of passing students over the given three years is the least? 

A E 

B F 

C A 

D B 

Answer: D 

 

Q.75 If cosec4θ = sec(60 − 2θ), then θ is equal to: 

A 25 

B 20 

C 15 

D 18 

Answer: C 

English 

Instructions For the following questions answer them individually 

Q.76 Select the most appropriate meaning of the underlined idiom in the given sentence. Raju was playing with fire when he made speeches against the management. 

A smoking a cigarette 

B taking a grave risk 

C provoking the crowd 

D lighting candles 

Answer: B 

 

Q.77 Given below are four jumbled sentences. Select the option that gives their correct order. 

A. The market which has remained clogged with vehicles was all clear for pedestrians. B. The road has even been marked with stripes demarcating space for hawkers. 

C. It is also lined with beautiful potted plants to give it a green look. 

D. Visitors to the busy Karol Bagh market in Delhi were in for a surprise. 

A DABC 

B ADBC 

C BDAC 

D DCBA 

Answer: A 

 

Q.78 Select the most appropriate synonym of the given word. ADEPT 

A unknown 

B kind-hearted 

C skilled 

D alone 

Answer: C 

 

Q.79 Select the most appropriate synonym of the given word. INITIATE 

A show 

B slow 

C sign 

D start 

Answer: D 

 

Q.80 Select the most appropriate option to fill in the blank. Following detailed deliberations, the meeting has been ______till next week 

A proposed 

B reviewed 

C adjourned 

D cancelled 

Answer: C 

 

Q.81 Select the most appropriate antonym of the given word. PROLONG 

A allow 

B shorten 

C increase 

D prevent 

Answer: B 

 

Q.82 Select the word which means the same as the group of words given. One who does not tire easily 

A indestructible 

B inflatable 

C indefatigable 

D indelible 

Answer: C 

 

Instructions 

In the following passage some words have been deleted. Fill in the blanks with the help of the alternatives given. Select the most appropriate option for each blank. 

Comprehension: He remembers the first time he saw the elephant. “This was in 2007 at a (1) ______ called Thanni Paarai. Chinna Thambi was (2) ______ water on himself at a waterbody. The (3) ______ was green; not the kind people would drink,” says Abraham. “After a nice shower, the elephant walked (4) ______ to a rock, lifted it, and drank from a (5)______ under it.” 

Q.83 Select the most appropriate option to fill in blank No.(1) 

A market 

B place 

C street 

D shop 

Answer: B 

 

Q.84 Select the most appropriate option to fill in blank No.(2) 

A sprinkling 

B wetting 

C raining 

D sparkling 

Answer: A 

 

Q.85 Select the most appropriate option to fill in blank No.(3) 

A water 

B milk 

C flower 

D leaf 

Answer: A 

 

Q.86 Select the most appropriate option to fill in blank No.(4) 

A above 

B over 

C behind 

D near 

Answer: B 

 

Q.87 Select the most appropriate option to fill in blank No.(5) 

A pond 

B tap 

C spring 

D tank 

Answer: C 

 

Instructions For the following questions answer them individually 

Q.88 In the sentence identify the segment which contains the grammatical error. There is many modes of travel to go to Agra but I prefer road travel. 

A prefer road travel. 

B Agra but I 

C There is many modes 

D to go to 

Answer: C 

 

Q.89 Select the most appropriate option to fill in the blank. Hima Das, the reigning world junior sprinter who ______ the national record, won the gold in the Federation Cup. 

A keeps 

B holds 

C plays 

D gets 

Answer: B 

 

Q.90 In the sentence identify the segment which contains the grammatical error. Having just taking a heavy lunch, she was not ready to have any fruit. 

A she was not ready 

B to have any fruit. 

C Having just 

D taking a heavy 

Answer: D 

 

Q.91 Select the most appropriate antonym of the given word. SCANTY 

A concise 

B small 

C precise 

D profuse 

Answer: D 

 

Q.92 Select the correctly spelt word. 

A repitition 

B aggrravate 

C accomplish 

D hieghten 

Answer: C 

 

Q.93 Select the correctly spelt word. 

A luxry 

B luxury 

C lugzury 

D luxery 

Answer: B 

 

Q.94 Select the correct passive form of the given sentence. They opened a new mall nearby last month. 

A A new mall was opened nearby last month. 

B A new mall will be opened nearby last month. 

C Nearby a new mall is opened last month. 

D A new mall can be open nearby last month. 

Answer: A

 

Q.95 Select the most appropriate option to substitute the underlined segment in the given sentence. If no substitution is required, select No improvement. When we went to the cinema yesterday, the film had already start. 

A No improvement 

B the film had already started. 

C the film have already start. 

D the film was already start. 

Answer: B 

 

Q.96 Given below are four jumbled sentences. Select the option that gives their correct order. 

A. When we can’t laugh at the same joke again, why should we cry over the same problems? B. One day he told them a joke and everyone roared with laughter. 

C. People came to a wise man to complain about the same problems every time. 

D. When he repeated the joke twice, nobody laughed anymore. 

A ADBC 

B DBCA 

C BACD 

D CBDA 

Answer: D 

 

Q.97 Select the most appropriate option to substitute the underlined segment in the given sentence. If no substitution is required, select No improvement. I am think that tomorrow I will take leave and stay at home. 

A I think that tomorrow I will take 

B I was thinking that the next day I will be taking 

C No improvement 

D I have thought that tomorrow I am taking 

Answer: A 

 

Q.98 Select the most appropriate meaning of the underlined idiom in the given sentence. The authorities have turned a deaf ear to all our requests. 

A presented 

B accepted 

C acknowledged 

D neglected 

Answer: D 

 

Q.99 Select the word which means the same as the group of words given. Incapable of being read 

A vague 

B eligible 

C unseen 

D illegible 

Answer: D 

 

Q.100 Select the correct active form of the given sentence. By whom were you taught Mathematics? 

A Whom are you teaching Mathematics? 

B Who will teach you Mathematics? 

C Who teaching you Mathematics? 

D Who taught you Mathematics? 

Answer: D 

SSC CGL Tier-I 12 June 2019 Shift-II Previous Year Paper

SSC CGL

(12 June 2019 Shift-II) 

Reasoning 

Instructions For the following questions answer them individually 

Q.1 Three of the following four number-pairs are alike in a certain way and one is different. Pick the number- pair that is different from the rest. 

A 13 : 171 

B 8 : 66 

C 7 : 51 

D 10 : 98 

Answer: D 

 

Q.2 In a code language, ARDOUS is written as 1184152119. How will SYSTEM be written as in that language? 

A 19251921513 

B 19251920514 

C 18251820513 

D 19251920513 

Answer: D 

 

Q.3 The sum of the current ages of Asma and her grandfather is 80 years. 10 years from now, Asmma’s age will be one-fourth of her grandfather’s age. What is Asma’s current age? 

A 12 years 

B 20 years 

C 10 years 

D 16 years 

Answer: C 

 

Q.4 Select the missing number from the given options. 

A 64 

B 16 

C 25 

D 36 

Answer: A 

 

Q.5 Arrange the following words in a logical and meaningful order. 

1. Hunger 2. Cry 3. Feeding 4. Child 5. Sleep 6. Mother 

A 4, 1, 2, 3, 5, 6 

B 4, 1, 3, 6, 5, 2 

C 4, 5, 2, 6, 3, 1 

D 4, 1, 2, 6, 3, 5 

Answer: D 

 

Q.6 Which number will replace the question mark (?) in the following series? 

118, 129, 141, 154, 168, ? 

A 184 

B 182 

C 181 

D 183 

Answer: D 

 

Q.7 Select the option that is related to the third number in the same way as the second number is related to the first number. 7 : 344 :: 11 : ? 

A 1331 

B 1332 

C 122 

D 121 

Answer: B 

 

Q.8 In a code language, INFORMATIVE is written as ROFNILEVITA. How will SUPERFICIAL be written as in that language? 

A LAICIGREPUS 

B USEPRFICAIL 

C REPUSELAICI 

D REPUSGLAICI 

Answer: C 

 

Q.9 Select the figure that will come next in the following figure series. 

Answer: D 

 

Q.10 How many triangles are there in the following figure ? 

A 30 

B 24 

C 26 

D 28 

Answer: D 

 

Q.11 Which two signs should be interchanged to make the following equation correct? 

24 ÷ 12 − 6 × 6 + 2 = 18 

A × and +

B + and ÷

C + and −

D ÷ and ×

Answer: B 

 

Q.12 Three different positions of the same dice are shown. Which symbol will be on the face opposite to the one having ‘&’ ? 

A + 

B # 

C $ 

D ^ 

Answer: B 

 

Q.13 In the given Venn diagram, the pentagon represents ‘Cricketers’, the square represents ‘Chess players’ and the circle represents ‘Clerks’. The numbers given in the diagram represent the number of persons in that particular category. 

How many cricketers are chess players but NOT clerks? 

A 36 

B 34 

C 22 

D 40 

Answer: C 

 

Q.14 Select the correct mirror image of the given figure when the mirror is placed to the right of the figure. 

Answer: D 

 

Q.15 Two statements are given, followed by three conclusions numbered I, II and III. Assuming the statements to be true, even if they seem to be at variance with commonly known facts, decide which of the conclusions logically follow(s) from the statements. Statements: 

All postcards are envelopes. 

No envelope is a paper. 

Conclusions: 

I. Some envelopes are postcards. 

II. No paper is an envelope. 

III. No postcard is a paper. 

A All the conclusions, I, II and III, follow 

B Only conclusions I and II follow 

C Only conclusions I and III follow. 

D Only conclusions II and III follow 

Answer: A 

 

Q.16 Which letter-cluster will replace the question mark (?) in the following series? 

PRT, TVX, ?, BDF, FHJ 

A XAB 

B XZB 

C XZC 

D YZB 

Answer: B 

 

Q.17 ‘Valuable’ is related to ‘Precious’ in the same way as ‘Dry’ is related to ‘ ……… ’ 

A Rough 

B Wet 

C Farm 

D Arid 

Answer: D 

 

Q.18 Three of the following four words are alike in a certain way and one is different. Pick the odd word out. 

A Ears 

B Nose 

C Eyes 

D Throat 

Answer: D 

 

Q.19 Select the missing number from the given options. 

A 12 

B 16 

C 36 

D 25 

Answer: B

 

Q.20 Three of the following four letter-clusters are alike in a certain way and one is different. Pick the odd one out. 

A SUXBG 

B OQTXC 

C BDGKP 

D JLKRW 

Answer: D 

 

Q.21 Select the figure in which the given figure is embedded. (rotation is not allowed). 

Answer: D 

 

Q.22 Which letter will replace the question mark (?) in the following series? 

P, U, R, W, T, 2, V, A,X 

A X 

B W 

C Z 

D Y 

Answer: D 

 

Q.23 Select the word-pair in which the two words are related in the same way as are the two words in the following word pair. Dermatology : Skin 

A Paediatrics : Lungs 

B Orthopaedics : Bones 

C Medicine : Treatment 

D Cardiology : Angiography 

Answer: B 

 

Q.24 Select the option that depicts how the given transparent sheet of paper would appear if it is folded at the dotted line. 

Answer: B 

 

Q.25 ‘A + B’ means ‘A is the brother of B’. 

‘A – B’ means ‘A is the wife of B’. 

‘A ×B’ means ‘A is the daughter of B’. 

‘A ÷ B’ means ‘A is the father of B’. 

If P + S × Q × R – T ÷ V ÷ U, then how is T related to P ? 

A Maternal grandmother 

B Paternal grandfather 

C Maternal grandfather 

D Paternal grandmother 

Answer: C 


General knowledge 

Instructions For the following questions answer them individually 

Q.26 How many International airports are there in India? 

A 15 

B 14 

C 17 

D 16 

Answer: C 

 

Q.27 Which of the following is the national bird of China? 

A Barn Owl 

B Robin 

C Red-crowned Crane 

D Parrot 

Answer: C 

 

Q.28 What is the chemical name of the deadly poison ‘cyanide’? 

A Prussic acid 

B Sulfuric acid 

C Hydrochloric acid 

D Nitric acid 

Answer: A 

 

Q.29 The river Ganga emerges from Gangotri Glacier and ends at ______. 

A Bay of Bengal 

B Arabian Sea 

C Indian Ocean 

D Pacific Ocean 

Answer: A 

 

Q.30 What happens when the fiscal deficit increases 

A prices increase 

B no direct impact on prices 

C prices remain constant 

D prices decrease 

Answer: B 

 

Q.31 All the tangible resources like raw materials and labor used in production process are called 

A Opportunity cost 

B Variable cost 

C Fixed cost 

D Real cost 

Answer: D 

 

Q.32 which year was the Indian super league launched in? 

A 2013 

B 2015 

C 2012 

D 2014 

Answer: A

 

Q.33 The term ______ is prefixed to scientific terms to describe something that is constant. 

A Photo 

B Iso 

C Mega 

D Quasi 

Answer: B 

 

Q.34 Who is the first Indian national to win the ‘Sri Lanka Ratna’ award? 

A Dileep Padgaonkar 

B Khushwant Singh 

C Vinod Mehta 

D Narasimhan Ram 

Answer: D 

 

Q.35 Who presides over the joint sitting of the Lok Sabha and the Rajya Sabha? 

A Deputy Speaker of Lok Sabha 

B Prime Minister 

C Vice President 

D Speaker of Lok Sabha 

Answer: D 

 

Q.36 The word “blood moon” is used for 

A Lunar Eclipse 

B Full moon 

C Solar Eclipse 

D Half moon 

Answer: A 

 

Q.37 Which country in the world has the largest number of international borders? 

A Nepal 

B Pakistan 

C India 

D China 

Answer: D 

 

Q.38 Who is the author of the book ‘Freedom from Fear: And Other Writings’? 

A Aung San Suu Kyi 

B Barak Obama 

C A P J Abdul Kalam 

D Nelson Mandela 

Answer: A 

 

Q.39 Which instrument is used to measure blood pressure? 

A Sphygmomanometer 

B Lactometer 

C Thermometer 

D Glucometer 

Answer: A 

 

Q.40 In which year was the University Grants Commission (UGC) formally established? 

A 1950 

B 1964 

C 1947 

D 1956 

Answer: D 

 

Q.41 Dolly, the first cloned living being created at Roslin Institute in Scotland was a _____. 

A cow 

B dog 

C sheep 

D cat 

Answer: C 

 

Q.42 Who among the following became the fastest Asian to cycle around the globe in 2018? 

A Deena Prince 

B Vedangi Kulkarni 

C Disha Srivastava 

D Swati Sani 

Answer: B 

 

Q.43 Mitra Mela was a revolutionary organisation founded by Veer Savarkar in the year ____ 

A 1900 

B 1873 

C 1864 

D 1856 

Answer: A 

 

Q.44 UN’s Economic and Social Council (ECOSOC) elected _________ to the 18-member committee ‘CESCR’ for the Asia Pacific seat in 2018. 

A Chandrashekhar Dasgupta 

B Shashi Tharoor 

C Preeti Saran 

D Arundhati Ghosh 

Answer: C 

 

Q.45 In which year did the Indian Premier League (IPL) start? 

A 2002 

B 2008 

C 2004 

D 2010 

Answer: B 

 

Q.46 Under which article of the Constitution of India can members of the Anglo Indian community be nominated to the Lok Sabha by the President? 

A 326 

B 331 

C 330 

D 342 

Answer: B 

 

Q.47 Burma became an independent sovereign republic in the year _____. 

A 1948 

B 1946 

C 1962 

D 1950 

Answer: A 

 

Q.48 Catriona Gray of _____ was crowned Miss Universe in 2018. 

A Brazil 

B Venezuela 

C Philippines 

D Spain 

Answer: C 

 

Q.49 Who among the following was given the title ‘Quaid-i-Azam’? 

A Mahatma Gandhi 

B Jawaharlal Nehru 

C Sardar Vallabhbhai Patel 

D Muhammad Ali Jinnah 

Answer: D 

 

Q.50 The traditional art of ‘Jamdani’ weaving originated in ______. 

A Nepal 

B Myanmar (Burma) 

C Bangladesh 

D China 

Answer: C 

 

Quantitative Aptitude 

Instructions For the following questions answer them individually 

Q.51 In △ABC,P is a point on BC such that BP : PC = 3 : 4 and Q is the midpoint of BP. Then ar( △ABQ): ar( △ABC) is equal to: 

A 3 : 8 

B 2 : 7 

C 1 : 4 

D 3 : 14 

Answer: D 

 

Q.52 The radii of two bases of a frustum of height 10.5cm is 5cm and 3cm. What is its volume in cm3 ( π = 22/7)?

A 552 

B 545 

C 539 

D 564 

Answer: C 

 

Q.53 Table shows the production of rice (in million tonnes) of three states over six years. 

What is the average production of rice in state A over the years (in million tonnes)? 

A 5.8 

B 6.1 

C 6 

D 5.9 

Answer: C 

 

Q.54 Two articles are sold for ₹ 4880, on one, the seller gained 22% and on the other he lost 20%. What is his overall gain or loss percentage, nearest to one decimal place? 

A 3.4% loss 

B 3.4% gain 

C 3.6% gain 

D 3.6% loss 

Answer: A 

 

Q.55 In a class of 60 students, 40% are girls. The average weight of the whole class is 59.2 kg and the average weight of the girls is 55 kg. What is the average weight of the boys? 

A 62 kg 

B 61 kg 

C 60 kg 

D 63 kg 

Answer: A 

 

Q.56 In a circle of radius 13cm, a chord is at a distance of 5cm from its centre. What is the length of the chord? 

A 24 cm 

B 18 cm 

C 12 cm 

D 20 cm 

Answer: A 

 

Q.57 The efficiencies of A, B and C are in the ratio of 2 : 3 : 5. Working together, they can complete a task in 6 days. In how many days will A alone complete 20% of that task? 

A 4 

B 8 

C 5 

D 6 

Answer: D 

 

Q.58 If 3sinθ = 4cosθ, then tan2 θ + sinθ − cosθ is equal to: 

A 88/45
B 2 

C 89/45

D 17/9

Answer: C 

 

Q.59 If √x + 1/√x= 2√2 , then x2 + 1/x2 is equal to: 

A 102 

B 98 

C 104 

D 100 

Answer: B 

 

Q.60 What is the value of x so that the seven digit number 6913 x 08 is divisible by 88? 

A 6 

B 4 

C 2 

D 8 

Answer: D

 

Q.61 An article is sold for ₹ 612 after successive discounts of 25% and 15%. What is the marked price of the article? 

A ₹960 

B ₹940 

C ₹1000 

D ₹980 

Answer: A 

 

Q.62 △ABC ∼ △RQP and PQ = 10 cm, QR =12 cm and RP = 18 cm. If ar( △ABC) : ar( △POR) = 4/9, then AB is equal to: 

A 20/3 cm 

B 8 cm 

C 9 cm 

D 12 cm 

Answer: B 

 

Q.63 Walking at ¾ of his usual speed, a person reaches his office 18 minutes later than the usual time. His usual time in minutes is: 

A 72 

B 45 

C 60 

D 54 

Answer: D 

 

Q.64 If a : b = 2 : 3, then (5a + 3b) : (6a — 2b) is equal to: 

A 3 : 2 

B 17 : 5 

C 10 : 7 

D 19 : 6 

Answer: D 

 

Q.65 Table shows the production of rice (in million tonnes) of three states over six years. 

If the total production in three states in all six years is represented by a pie-chart, what is the central angle of the sector representing production in the year 2014, (to nearest whole number)? 

A 65 

B 61 

C 59 

D 63 

Answer: D 

 

Q.66 If cosec3θ = sec(20 + 2θ), then θ is equal to: 

A 14 

B 20 

C 30 

D 15 

Answer: A 

Instructions Table shows the production of rice (in million tonnes) of three states over six years. 

Q.67 What is the percentage increase in the production of rice in B from 2014 to 2016? 

A 17.75 

B 17.25 

C 18.75 

D 18.25 

Answer: C 

 

Q.68 What is the ratio of the production of rice in all three states in the year 2014 to that in 2016? 

A 89 : 100 

B 85 : 102 

C 85 : 103 

D 87 : 100 

Answer: D 

 

Instructions For the following questions answer them individually 

Q.69 The value of 7.5 + (5.4 ÷ 4.5 × 2) − 8 × 4 ÷ 3.2: 

A -0.2 

B 0.1 

C -0.1 

D 0.2 

Answer: C 

 

Q.70 The value of sin2 20 + sin2 70 − tan2 45 + sec60is equal to: 

A 2 

B 1 

C 2.5 

D 3 

Answer: A 

 

Q.71 The arc ABC of a circle with centre O subtends 132 at the centre. The chord AB is extended to the point P. The angle ∠CBP is equal to 

A 48

B 66

C 76

D 68

Answer: B 

 

Q.72 The difference between compound interest and simple interest on ₹ x at 8% per annum for 2 years is ₹ 48. What is the value of x? 

A 7500 

B 7400 

C 8000 

D 7800 

Answer: A 

 

Q.73 If a + b + c = 6 and a3 + b3 + c3 − 3abc= 126, then ab + bc + ca is equal to: 

A 12 

B 8 

C 5 

D 6 

Answer: C 

 

Q.74 If a + b = 5 and ab = 3, then(a3 + b3) is equal to: 

A 75 

B 80 

C 70 

D 65 

Answer: B 

 

Q.75 The price of sugar is increased by 24%. A person wants to increase his expenditure by 15% only. By what percentage, correct to one decimal place, should he reduce his consumption? 

A 7.1 

B 7.3 

C 6.9 

D 7.5 

Answer: B 

 

English 

Instructions For the following questions answer them individually 

Q.76 Select the correctly spelt word. 

A qiete 

B quiet 

C queit 

D quiete 

Answer: B 

 

Q.77 Select the most appropriate synonym of the given word. ASSIST 

A help 

B create 

C change 

D mend 

Answer: A 

 

Q.78 Select the word which means the same as the group of words given. That which cannot be heard 

A hidden 

B inaudible 

C slight 

D invisible 

Answer: B 

 

Q.79 Choose the option that is the passive form of the sentence. A campus fire in California caused the death of at least twenty-three persons. 

A The death of at least twenty three persons will be caused in a campus fire in California. 

B The death of at least twenty three persons was caused by a campus fire in California. 

C At least twenty-three person’s death was caused in a campus fire in California. 

D The death of at least twenty three persons caused a campus fire in California. 

Answer: B 

 

Q.80 Identify the segment in the sentence, which contains the grammatical error. “Unless you did not do your homework you will be punished,” said the teacher. 

A Unless you did not 

B said the teacher. 

C do your homework 

D you will be punished 

Answer: A 

 

Q.81 Select the most appropriate synonym of the given word INTRICATE 

A complex 

B complete 

C connected 

D colorful 

Answer: A 

 

Q.82 Select the most appropriate antonym of the given word. ABSURD 

A sensible 

B selfish 

C sentimental 

D sensitive 

Answer: A 

 

Q.83 Select the most appropriate option to fill in the blank. When you want to expand your vocabulary the best thing to do is to relate a known word with an ______ one and guess the meaning from the context. 

A unnecessary 

B unfamiliar 

C essential 

D unclear 

Answer: B 

 

Q.84 Given below are four jumbled sentences. Select the option that gives their correct order. 

A. If the mixture becomes too thick add more milk. 

B. Finally, add sugar and nuts, and your kheer is ready. 

C. Boil milk in a heavy-bottomed pan and add rice. 

D. Cook for about twenty minutes stirring every once in a while till the mixture thickens. 

A DACB 

B BADC 

C ADCB 

D CDAB 

Answer: D 

 

Q.85 Select the alternative that will improve the underlined part of the sentence in case there is no improvement select “No improvement” I am very much pleased to see you here today. 

A No improvement 

B very pleasing 

C very pleased 

D too much pleased 

Answer: C 

 

Q.86 Select the correctly spelt word. 

A restaurent 

B roberry 

C meazure 

D leisure 

Answer: D 

 

Q.87 Select the most appropriate option to fill in the blank. In order to ______ to a new place you may need to adjust to the ways of that culture. 

A avoid 

B adapt 

C adhere 

D adopt 

Answer: B 

 

Instructions: In the following passage some words have been deleted. Fill in the blanks with the help of the alternatives given. Select the most appropriate option for each blank. 

Comprehension: Although a wild elephant, Chinna Thambi did not grow up entirely in the wild. For over the last (1) ______he has had easy (2)______ to food at Thadagam, a village surrounded by the Western Ghats near Coimbatore. Brick (3) ______ tempted him with (4)______ of water and he loved the palm pith that was used as (5)______ for baking bricks. 

Q.88 Select the most appropriate option to fill in blank No.(1) 

A annual 

B summer 

C festival 

D decade 

Answer: D 

 

Q.89 Select the most appropriate option to fill in blank No.(2) 

A entry 

B access 

C way 

D approach 

Answer: B 

 

Q.90 Select the most appropriate option to fill in blank No.(3) 

A kilns 

B jar 

C tub 

D store 

Answer: A 

 

Q.91 Select the most appropriate option to fill in blank No.(4) 

A load 

B many 

C much 

D plenty 

Answer: D 

 

Q.92 Select the most appropriate option to fill in blank No.(5) 

A logs 

B fuel 

C matter 

D fodder 

Answer: B 

 

Instructions For the following questions answer them individually 

Q.93 Select the most appropriate meaning of the underlined idiom in the given sentence. Throughout his speech the crowd was all ears. 

A making a lot of noise 

B talking loudly 

C very attentive 

D covering their ears 

Answer: C 

 

Q.94 Choose the option that is the active form of the sentence. It was decided by the members that the report would be placed before the Chairman for his comments. 

A Did the members decide to place the report before the Chairman? 

B The Chairman’s comments were to be placed on the report of the members. 

C The members decided to place the report before the Chairman for his comments. 

D The Chairman decided to place the report before the members. 

Answer: C

 

Q.95 Select the alternative that will improve the underlined part of the sentence in case there is no improvement select “No improvement”. The flowers smell so sweetly that I want to pluck them. 

A smell so sweet 

B No improvement 

C smelling so sweetly 

D will smell so sweet 

Answer: A 

 

Q.96 Identify the segment in the sentence, which contains the grammatical error. The reduction in the cost of education due to an increase in subsidies offer by the government is also seen as a reason to get more educated. 

A The reduction in the cost of education 

B is also seen as a reason 

C due to an increase in subsidies 

D offer by the government 

Answer: D 

 

Q.97 Select the most appropriate antonym of the given word. ACQUIT 

A clear 

B evict 

C convict 

D forgive 

Answer: C 

 

Q.98 Given below are four jumbled sentences. Select the option that gives their correct order. 

A. This network of stories is known in academic circles as ‘fiction’ or ‘imagined realities’. 

B. However, an imagined reality is not a lie. 

C. Over the years, people have woven an incredibly complex network of stories. 

D. Within this network fiction not only exists but also accumulates immense power. 

A CADB 

B DBCA 

C ADBC 

D DBAC 

Answer: A 

 

Q.99 Select the most appropriate meaning of the underlined idiom in the given sentence. Since he secured the first rank Sudhir has become swollen-headed. 

A well-connected 

B unwell 

C famous 

D conceited 

Answer: D 

 

Q.100 Select the word which means the same as the group of words given. Incapable of being corrected 

A inviolable 

B impossible 

C incredible 

D incorrigible 

Answer: D 

SSC CGL Tier-I 12 June 2019 Shift-I Previous Year Paper

SSC CGL 

(12 June 2019 Shift-I) 

Reasoning 

Instructions For the following questions answer them individually 

Q.1 In a code language, YOGHURT is written as 251578211820. How will DEVELOP be written as in that language? 

A 45225121515 

B 45215121516 

C 45225121516 

D 45225111516 

Answer: C 

 

Q.2 Three of the following four words are alike in a certain way and oneis different. Pick the odd word out. 

A Examine 

B Deliberate 

C Inquire 

D Explore 

Answer: B 

 

Q.3 Which number will replace the question mark (?) in the following series? 

115, ?, 134, 145, 157, 170 

A 122 

B 124 

C 127 

D 125 

Answer: B 

 

Q.4 ‘A + B’ means ‘A is the sister of B’. ‘A – B’ means ‘A is the brother of B’. ‘A ×B’ means ‘A is the mother of B’. ‘A ÷ B’ means ‘A is the father of B’. If V + S × Q – P ÷ T + R × U, then how is R related to S ? 

A Daughter 

B Grandson 

C Maternal grandmother 

D Grand-daughter 

Answer: D 

 

Q.5 Three different positions of the same dice are shown. Which number will be on the face opposite to the one having 2 ?

A 3 

B 5 

C 4 

D 6 

Answer: C 

 

Q.6 How many triangles are present in the following figure ?

A 19 

B 20 

C 18 

D 17 

Answer: A 

 

Q.7 Two statements are given, followed by three conclusions numbered I, II and III. Assuming the statements to be true, even if they seem to be at variance with commonly known facts, decide which of the conclusions logically follow(s) from the statements. Statements: 

All drums are sticks. 

Some drums are boxes. 

Conclusions: 

I Some boxes are sticks. 

II. Some sticks are drums. 

III. All sticks are drums. 

A Only conclusions I and III follow. 

B All the conclusions, I, II and III follow 

C Only conclusions II and III follow 

D Only conclusions I and II follow 

Answer: D 

 

Q.8 Which two signs should be interchanged to make the following equation correct? 

4 × 5 − 24 ÷ 12 + 8 = 14

A ÷ and ×

B × and +

C + and −

D + and ÷

Answer: C 

 

Q.9 Select the option that depicts how the given transparent sheet of paper would appear if it is folded at the dotted line. 

Answer: D 

 

Q.10 Arrange the following words in a logical and meaningful order. 

1. Writing 2. Book 3. Seller 4. Idea 5. Feedback 6. Reader 

A 4, 1, 2, 6, 3, 5 

B 4, 1, 2, 3, 6, 5 

C 4, 1, 2, 6, 5, 3 

D 4, 1, 3, 6, 5, 2 

Answer: B 

 

Q.11 Which letter will replace the question mark (?) in the following series? X, R, ?, I, F, D, C 

A Q 

B N 

C M 

D O 

Answer: C 

 

Q.12 Select the figure in which the given figure is embedded. (Rotation is not allowed). 

Answer: B 

 

Q.13 In the given Venn diagram, the circle represents ‘businessmen’, the triangle represents ‘landlords’ and the rectangle represents ‘income tax payers’. The numbers given in the diagram represent the number of persons in that particular category. 

How many businessmen are tax payers but NOT landlords? 

A 22 

B 8 

C 17 

D 19 

Answer: A 

 

Q.14 Select the correct mirror image of the given figure when the mirror is placed to the right of the figure. 

Answer: A 

 

Q.15 Select the word-pair in which the two words are related in the same wayas are the two words in the following word pair. Suitcase : Cloak Room 

A Salary : Credit 

B Bag : Luggage 

C Chair : Furniture 

D Money : Bank 

Answer: D 

 

Q.16 In a code language, PERMANENT is written as EPMRBENTN. How will TECHNICAL be written as in that language? 

A ETHCPCILA 

B ETHCOCILA 

C ETCHOCILA 

D TEHCOCIAL 

Answer: B 

 

Q.17 Select the missing number from the below options. 

A 55 

B 53 

C 54 

D 52 

Answer: A 

 

Q.18 Three of the following four number-pairs are alike in a certain way and oneis different. Pick the number- pair that is different from the rest. 

A 7 : 48 

B 13 : 170 

C 11 : 120 

D 5 : 24 

Answer: B 

 

Q.19 Select the option that is related to the third number in the same way as the second number is related to the first number. 9 : 90 :: 12 : ? 

A 156 

B 152 

C 160 

D 150 

Answer: A

 

Q.20 Select the figure that will come next in the following figure series. 

Answer: A 

 

Q.21 The sum of the current ages of Vishal and Armaan is 70 years. 5 years ago, Vishal was twice as old as Armaan. What is Armaan’s current age? 

A 10 years 

B 45 years 

C 25 years 

D 20 years 

Answer: C 

 

Q.22 Three of the following four letter-clusters are alike in a certain way and one is different. Pick the odd one out. 

A JKNRV 

B PQTXC 

C GHKOT 

D UVYCH 

Answer: A 

 

Q.23 Select the missing number from the below options. 

A 14 

B 10 

C 12 

D 11 

Answer: B 

 

Q.24 ‘Political Science’ is related to ‘Social Science’ in the same way as ‘Chemistry’ is related to ‘ ………’ 

A Humanities 

B Elements 

C Science 

D Equations 

Answer: C 

 

Q.25 Which letter-cluster will replace the question mark (?) in the following series? FKP, HNT, JQX, LTB, ? 

A NWF 

B NXF 

C NWG 

D MWF 

Answer: A 

 

General knowledge 

Instructions For the following questions answer them individually 

Q.26 What is the dominant chemical present in detergent powder? 

A Calcium carbonate 

B Sodium alkyl sulphate 

C Hydrochloric acid 

D Sodium carbonate 

Answer: D 

 

Q.27 Rajat has hypermetropia. What type of lens will the ophthalmologist recommend to correct his vision? 

A Convex 

B Progressive 

C Bifocal 

D Concave 

Answer: A 

 

Q.28 What effect will a decrease in demand and an increase in supply have on equilibrium price? 

A Sometimes price will rise and sometimes it will fall 

B Equilibrium price will fall 

C Equilibrium price will rise 

D Equilibrium price will be constant 

Answer: B 

 

Q.29 Name the tissue that transports food to various parts of a plant. 

A Parenchyma 

B Sclerenchyma 

C Xylem 

D Phloem 

Answer: D 

 

Q.30 To which of the following Indian states does the tribe of ‘Nyishi’ belong? 

A Chhattisgarh 

B Tamil Nadu 

C Arunachal Pradesh 

D Bihar 

Answer: C 

 

Q.31 During World War II, the Battles of Kohima and Imphal were fought in the year _____. 

A 1942 

B 1944 

C 1945 

D 1943 

Answer: B 

 

Q.32 Which article of the Constitution of India provides that each Indian state will have a governor? 

A Article 151 

B Article 154 

C Article 152 

D Article 153 

Answer: D

 

Q.33 Which Indian badminton player was runner-up at the 2019 Swiss Open tournament? 

A Kidambi Srikanth 

B Sai Praneeth 

C Parupalli Kashyap 

D Chetan Anand 

Answer: B 

 

Q.34 Which of the following is called the ‘popular chamber’? 

A Gram Sabha 

B Rajya Sabha 

C State Assembly 

D Lok Sabha 

Answer: D 

 

Q.35 The term ‘Dolphin Kick’ is associated with which sport? 

A Football 

B Rugby 

C Swimming 

D Cricket 

Answer: C 

 

Q.36 Which Indian batsman was the first to hit six consecutive sixes in first-class cricket? 

A Ravi Shastri 

B Virat Kohli 

C Sachin Tendulkar 

D Sunil Gavaskar 

Answer: A 

 

Q.37 _____ has been established as the annual ‘International Day of Happiness’. 

A 15th January 

B 31st March 

C 20th March 

D 23rd February 

Answer: C 

 

Q.38 Dashain’ is the grandest festival of ______. 

A Nepal 

B Bangladesh 

C Bhutan 

D Sri Lanka 

Answer: A 

 

Q.39 Which of the following is the highest mountain peak in Maharashtra? 

A Taramati 

B Anjaneri 

C Salher 

D Kalsubai Shikhar 

Answer: D 

 

Q.40 Which of the following Indian film-makers was given the title of ‘Ambassador of Interlaken’ in 2011 at Switzerland? 

A Subhash Ghai 

B Yash Johar 

C Yash Chopra 

D Raj Kapoor 

Answer: C 

 

Q.41 Who was the last ruler of the Vaghela Dynasty of Gujarat after whose defeat the kingdom was passed to Alauddin Khilji? 

A Rama 

B Arjuna Deva 

C Karandev 

D Saranga Deva 

Answer: C 

 

Q.42 Who is the author of the book ‘India Shastra: Reflections on the Nation in our Time’? 

A Manmohan Singh 

B Narendra Modi 

C A P J Abdul Kalam 

D Shashi Tharoor 

Answer: D 

 

Q.43 Which of the following countries is the largest producer of wheat in the world? 

A India 

B Bangladesh 

C Myanmar (Burma) 

D China 

Answer: D 

 

Q.44 What was India’s rank among 156 countries in the Global Happiness Index 2019? 

A 132nd 

B 124th 

C 137th 

D 140th 

Answer: D 

 

Q.45 Blue litmus paper turns ____ on contact with an acidic solution. 

A green 

B red 

C yellow 

D brown 

Answer: B 

 

Q.46 _____ is well-known for the golden beautification of the Harmandir Sahib Gurdwara in Amritsar, famously known as the Golden Temple. 

A Charat Singh 

B Maha Singh 

C Ranjit Singh 

D Duleep Singh 

Answer: C 

 

Q.47 What is the distinctive characteristic of ‘marsupials’? 

A They hibernate in winter 

B They lay eggs 

C They carry young ones in pouches 

D They migrate from one place to another 

Answer: C 

 

Q.48 Which state of India has the longest mainland coastline? 

A Odisha 

B Kerala 

C Maharashtra 

D Gujarat 

Answer: D 

 

Q.49 A substantial increase in capital expenditure or revenue deficit leads to ______. 

A Budgetary Deficit 

B Fiscal Deficit 

C Primary Deficit 

D Revenue Deficit 

Answer: B 

 

Q.50 In the year _________, the Maratha Empire ceased to exist with the surrender of the Marathas to the British, ending the Third Anglo-Maratha War. 

A 1818 

B 1792 

C 1811 

D 1806 

Answer: A 

 

Quantitative Aptitude

Instructions For the following questions answer them individually 

Q.51 Two articles are sold for ₹ 4,956 each. On one,the seller gains 18% and on the other he loses 16%. What is his overall gain or loss percent to nearest one decimal place?? 

A 1.9% loss 

B 1.9% gain 

C 2.1% gain 

D 2.1% loss 

Answer: A 

 

Q.52 The following table shows the percentage distribution of students in various disciplines from five different colleges. 

What is the average number of students from the science discipline of all the colleges taken together? 

A 3748 

B 3762 

C 3642 

D 3724 

Answer: D 

 

Q.53 In a circle with centre O, AB is a diameter and CD is a chord such that ABCD is a trapezium. If ∠BAC = 28, then ∠CAD is equal to: 

A 32

B 62

C 34

D 28

Answer: C 

 

Q.54 In △ABC,P is a point on BC such that BP : PC = 2 : 3 and is the midpoint of BP. Then ar( △ABQ): ar( △ABC)is equal to: 

A 1:5 

B 1:4 

C 2:3 

D 2:5 

Answer: A 

 

Q.55 The following table shows the percentage distribution of students in various disciplines from five different colleges. 

If the data of the total students’ college wise, is represented by a pie-chart, what is the central angle of the sector representing college E (to nearest whole number)? 

A 73 

B 78 

C 79 

D 75 

Answer: D 

 

Q.56 The value of: 3.8 + (8.2 ÷ 4.1 × 2) − 4 × 3 ÷ 1.2

A 1.2 

B -2.2 

C 2.2 

D -1.2 

Answer: B 

 

Q.57 The price of sugar is increased by 17%. A person wants to increase his expenditure by 7% only. By what percentage, correct to one decimal place, should he reduce his consumption? 

A 8.3% 

B 8.1% 

C 8.7% 

D 8.5% 

Answer: D 

 

Q.58 △ABC∼△RQP and PQ = 10 cm, QR = 12 cm and RP = 16 cm. If ar(△PQR): ar (△ABC) =⁹/₄, then BC is equal to: 

A 8 cm 

B 6 cm 

C ²⁰/₃cm 

D ³²/₃cm 

Answer: C 

 

Q.59 If a + b + c = 6 and ab + bc + ca = 5, then a3+b3+c3-3abc is equal to: 

A 108 

B 126 

C 98 

D 116 

Answer: B 

 

Q.60 If a : b = 2 : 3, then (5a – 2b) : (5a+ 2b) is equal to: 

A 2:7 

B 3:7 

C 1:3 

D 1:4 

Answer: D

 

Q.61 If tan4θ = cot(40 − 2θ),then θ is equal to: 

A 25 

B 30 

C 20 

D 35 

Answer: A 

 

Q.62 A is 50% more efficient than B and C is 40% less efficient than B. Working together, they can complete a task in 20 days. In how many days will C alone complete 30% of that task ? 

A 31 

B 29 

C 33 

D 35 

Answer: A 

 

Q.63 A train without stoppage travels with an average speed of 80 km/h and with stoppage,it travels with an average speed of 64 km/h. For how many minutes does the train stop on an average per hour? 

A 08 

B 14 

C 12 

D 10 

Answer: C 

 

Q.64 The following table shows the percentage distribution of students in various disciplines from five different colleges. 

The number of students from the discipline of Economics from college B is approximately what percentage of the number of students from the discipline of Science from the college C? 

A 56 

B 59 

C 58 

D 61 

Answer: B 

 

Q.65 The value of: sin44/cos46 + sin2 60 − cos2 45 + sec60 is equal to: 

A 13/4

B 11/3

C 11/4

D 7/4

Answer: A 

 

Q.66 If cosθ = ⅘ , then sin2 θcosθ + cos2 θsinθ is equal to: 

A 16/25

B 84/125 

C 14/25

D 82/125 

Answer: B 

 

Q.67 If (a + b) = 6 and ab = 8, then (a3+b3) is equal to: 

A 216 

B 72 

C 144 

D 108 

Answer: B 

 

Q.68 Chords AB and CD of a circle intersect at a point P inside the circle. If AB = 10 cm, AP = 4 cm and PC = cm, then CD is equal to: 

A 4.8 cm 

B 9.8 cm 

C 7.8 cm 

D 6.8 cm 

Answer: B 

 

Q.69 The radii of the two circular faces of the frustum of a cone of height 21 cm are 5 cm and 3 cm. What is its volume in cm3 (π = ²²/₇ ) 

A 1020 

B 1058 

C 1078 

D 1025 

Answer: C 

 

Q.70 What is the value of x so that the seven digit number 91876×2 is divisible by 72? 

A 7 

B 3 

C 2 

D 5 

Answer: B 

 

Q.71 The following table shows the percentage distribution of students in various disciplines from five different colleges. 

What is the percentage of students from the discipline of Mathematics for colleges A and C taken together, (nearest to one decimal place)? 

A 37.2 

B 36.9 

C 37.5 

D 36.7 

Answer: D 

 

Q.72 The difference between the compound interest and simple interest on ₹x at 12% per annum for 2 years is ₹43.20. What is the value of x? 

A 3,000 

B 2,500 

C 2,800 

D 2,400 

Answer: A 

 

Q.73 An article is sold for ₹ 547.40 after successive discounts of 30% and 15%. What is the marked price of the article? 

A ₹940 

B ₹900 

C ₹920 

D ₹960 

Answer: C 

 

Q.74 Ina class of 60 students, 40% are girls. The average weight of the boys is 62kg and that of the girls is 55kg. What is the average weight of the whole class? 

A 59.2kg 

B 58.6kg 

C 58.8kg 

D 59kg 

Answer: A 

 

Q.75 If √x + 1/√ x= 2√2 , then x2 + 1/x2 is equal to: 

A 36 

B 34 

C 32 

D 64 

Answer: B 

 

English 

Instructions For the following questions answer them individually 

Q.76 Given below are four jumbled sentences. Select the option that gives their correct order. 

A. So, I had thoughts of setting up an Indian restaurant there with my wife’s support. 

B. It took us two months to redesign the place to suit our needs. 

C. One of the things I really missed when I set up home in Maryla nd, was a restaurant that served authentic Indian food. 

D.I decided to pursue this idea seriously and bought an old building in the downtown. 

A CBAD 

B CADB 

C DACB 

D BCDA 

Answer: B 

 

Q.77 Select the word which means the same as the group of words given. A state of perfect balance 

A equilibrium 

B equinox 

C equilateral 

D equivalent 

Answer: A 

 

Q.78 In the sentence identify the segment which contains the grammatical error. Not complying by any of the laws can land you into serious trouble. 

A can land you 

B into serious trouble 

C any of the laws 

D Not complying by 

Answer: D 

 

Q.79 Given below are four jumbled sentences. Select the option that gives their correct order. 

A. Lyrics help in creating a distictive narrative, some conventions of which have been carried over from the talkies era. 

B. Thus, songs have outli ved films in people’s memories. 

C. However, songs seem to have acquired a musical gram mar of their own, establishing an emotional chord with the listeners. 

D. In popular Indian cinema, lyrics a re to music what the heart is to the body. 

A ADBC 

B ABCD 

C DCBA 

D DACB 

Answer: D 

 

Q.80 Select the most appropriate synonym of the given word. DEVOUT 

A revered 

B pious 

C respectable 

D loyal 

Answer: B 

 

Q.81 Select the correctly spelt word. 

A exilerate 

B exilarate 

C exhilarate 

D exhilerate 

Answer: C 

 

Q.82 Select the most appropriate option to fill in the blank. I sat ______ my life as nothing seemed to be working for me. 

A invoking 

B cursing 

C tormenting 

D blessing 

Answer: B 

 

Q.83 In the sentence identify the segment which contains the grammatical error. Raja Ravi Varma was one of the first artist who tried to create a style that was both modern and traditional. 

A to create a style that was 

B Raja Ravi Varma was 

C one of the first artist 

D both modern and traditional 

Answer: C 

 

Q.84 Select the most appropriate synonym of the given word. RENOWN 

A wisdom 

B conceit 

C fame 

D obscurity 

Answer: C 

 

Q.85 Select the most appropriate antonym of the given word. FOREIGN 

A rustic 

B rural 

C native 

D Indian 

Answer: C 

 

Q.86 Select the word which means the same as the group of words given. One who walks in sleep 

A pedestrian 

B philanthropist 

C somnambulist 

D omnipotent 

Answer: C 

 

Instructions In the following passage some words have been deleted. Fill in the blanks with the help of the alternatives given. Select the most appropriate option for each blank. 

Comprehension: English is not a language that (1)______ in India. But it is used extensively in (2)______country. India comes second on the list of countries (3)______most English speakers,albeit not as the first language. English is more (4)______than any single Indian language! It has managed to (5)______the entire world. 

Q.87 Select the most appropriate option to fill in blank No.(1) 

A originated 

B developed 

C created 

D derived 

Answer: A 

 

Q.88 Select the most appropriate option to fill in blank No.(2) 

A one 

B an 

C a 

D the 

Answer: D 

 

Q.89 Select the most appropriate option to fill in blank No.(3) 

A by 

B with 

C from 

D among 

Answer: B 

 

Q.90 Select the most appropriate option to fill in blank No.(4) 

A prevalent 

B frequented 

C extended 

D comprehensive 

Answer: A 

 

Q.91 Select the most appropriate option to fill in blank No.(5) 

A saturate 

B invade 

C pervade 

D persuade 

Answer: C 

 

Instructions For the following questions answer them individually 

Q.92 Select the most appropriate meaning of the underlined idiom in the given sentence. Many people join politics to feather their own nest. 

A make others’ life comfortable 

B serve their country 

C utilize black money 

D promote their own interest 

Answer: D 

 

Q.93 Select the correct passive form of the given sentence. Credit cards are replacing cash transactions. 

A Cash transactions are being replaced by credit cards. 

B Cash transactions had been replaced by credit cards. 

C Cash transactions are replaced by credit cards. 

D Cash transactions have been replaced by credit cards. 

Answer: A 

 

Q.94 Select the correctly spelt word. 

A preportion 

B propotion 

C proportion 

D proporsion 

Answer: C

 

Q.95 Select the most appropriate meaning of the underlined idiom in the given sentence. The man heaved a sigh of relief when he was sure he was out of the woods. 

A out of danger 

B released from prison 

C out of the forest 

D discharged from hospital 

Answer: A 

 

Q.96 Select the most appropriate option to fill in the blank. The roads at 15000 feet are not easily navigable and the air is ______ and freezing. 

A exalted 

B rarefied 

C intensified 

D elevated 

Answer: B 

 

Q.97 Select the most appropriate antonym of the given word. ASCENT 

A distant 

B depression 

C descent 

D decent 

Answer: C 

 

Q.98 Select the most appropriate option to substitute the underlined segment in the given sentence. If no substitution is required, select No improvement. I have not saw him since I last leave the town. 

A see him since I last left 

B seen him for I last left 

C seen him since I last left 

D No improvement 

Answer: C 

 

Q.99 Select the correct active form of the given sentence. We were given very little time to prepare the presentation. 

A They give us very little time to prepare the presentation 

B They will give us very little time to prepare the presentation. 

C They have given us very little time to prepare the presentation. 

D They gave us very little time to prepare the presentation. 

Answer: D 

 

Q.100 Select the most appropriate option to substitute the underlined segment in the given sentence. If no substitution is required, select No improvement. Ishwarchand Vidyasagar use the ancient text to suggestion that widows could remarry. 

A used the ancient texts suggests 

B used the ancient texts to suggest 

C use the ancient texts for suggestion 

D No improvement 

Answer: B 

SSC CGL Tier-I 13 June 2019 Shift-III Previous Year Paper

SSC CGL 

(13 June 2019 Shift-III) 

Reasoning 

Instructions For the following questions answer them individually 

Q.1 Select the figure that will come next in the following figure series. 

Answer: D 

 

Q.2 ACCIDENT is related to TNEDICCA in the same way as PASSENGER is related to *_ 

A REGMESSAP 

B RGENESSAP 

C REGNESSAP 

D REGNSESAP 

Answer: C 

 

Q.3 Select the letter-cluster that is related to the third letter-cluster in the same way that the second letter- cluster is related to the first letter-cluster. AFKP : BGLQ :: GLQV: ? 

A HMPW 

B HKRW 

C HMRW 

D HNRW 

Answer: C 

 

Q.4 Three of the following four number-pairs are alike in a certain way and one is different. Pick the odd number-pair out. 

A 123 : 11 

B 144 : 12 

C 225 : 15 

D 196 : 14 

Answer: A 

 

Q.5 Select the term that will come next in the following series. 

11, 13, 17, 23, 31, 41, 53, 67, 83, ? 

A 100 

B 101 

C 110 

D 97 

Answer: B 

 

Q.6 Select the word-pair in which the two words are related in the same way as the two words in the following word-pair. School : Student:: ……… : ……… 

A College : Teacher 

B Court : Judge 

C Hospital : Patient 

D Hotel : Chef 

Answer: C 

 

Q.7 In a code language, FRIEND is written as GQJDOC. How will PEACE bewritten in that language? 

A ODBBF 

B QDBDF 

C QDBBF 

D QFBBF 

Answer: C 

 

Q.8 If ‘J’ is coded as ’20’ and ‘BAT” is coded as ’46’ then how will ‘CRICKET’ be coded? 

A 138 

B 158 

C 142 

D 140 

Answer: A 

 

Q.9 Select the option that is related to the third number in the same way as the second number is related to the first number. 19 : 400 :: 24 : ………. 

A 566 

B 676 

C 652 

D 625 

Answer: D 

 

Q.10 Which two signs should be interchanged to make the following equation correct? 

20 ÷ 20 + 20 − 25 × 25 = 419 

A ÷ and × 

B + and ÷ 

C + and − 

D × and − 

Answer: A 

 

Q.11 Select the option that will fill in the blank and complete the given series.

0, 7, 26, 63, 124, 215, 342, 511, 728, ……… 

A 996 

B 1001 

C 999 

D 1000 

Answer: C 

 

Q.12 Select the option that will fill in the blank and complete the given series.

2, 5, 10, 17, 26, 37, ……… , 65, 82, 101 

A 54 

B 48 

C 51 

D 50 

Answer: D 

 

Q.13 Three of the following four words are alike in a certain way and one is different. Pick the odd one out. 

A Terminology 

B Cardiology 

C Pedology 

D Seismology 

Answer: A 

 

Q.14 If you wrote down all the numbers from 1 to 99, then how many times would you have written ‘7‘? 

A 19 

B 20 

C 21 

D 11 

Answer: B 

 

Q.15 Choose the Venn diagram which best illustrates the relationship among the three given classes: Fruit, Vegetable, Apple 

Answer: A 

 

Q.16 Select the figure in which the given figure is embedded. 

Answer: C 

 

Q.17 Two statements are given, followed by three conclusions numbered I, II and III. Assuming the statements to be true, even if they seem to beat variance with commonly known facts, decide which of the conclusions logically follow(s) from the statements. Statements: 

No grass is a flower. 

All flowers are trees. 

Conclusions: 

I. Some trees are flowers. 

II. Some trees are grasses. 

III. No tree is a grass. 

A Either conclusions II or III, and I follow 

B Only conclusion I follows 

C Only conclusion III follows 

D Either conclusion II or III follows 

Answer: A 

 

Q.18 Given here is a square transparent sheet with a pattern on it. How would the pattern appear when the transparent sheet is folded on the dotted line? 

Answer: B 

 

Q.19 A cube coloured pink on all faces is cut into 27 small cubes of equal sizes. How many cubes are painted on one face only? 

A 3 

B 6 

C 8 

D 4 

Answer: B

 

Q.20 How many triangles are there in the following figure? 

A 10 

B 12 

C 15 

D 9 

Answer: C 

 

Q.21 Select the correct mirror image of the following word when the mirror is placed to the right of the word. HINT

Answer: A 

 

Q.22 Arrange the following words in a logical and meaningful order.  

1. Rajasthan 2. India 3. Jaipur 4. North India 5. Asia 

A 1, 3, 4, 2, 5 

B 3, 1, 4, 2, 5 

C 3, 1, 2, 4, 5 

D 1, 3, 2, 4, 5 

Answer: B 

 

Q.23 Three of the following four letters are alike in a certain way and oneis different. Pick the odd one out. 

A U 

B I 

C E 

D V 

Answer: D 

 

Q.24 Introducing Kavi, Veena said, “She is the sister of the son of the wife of my husband”. How is Veena related to Kavi? 

A Sister 

B Daughter 

C Aunt 

D Mother 

Answer: D 

 

Q.25 Select the combination of letters that when sequentially placed in the gaps of the given letter series will complete the series _c_bd_cbcda_a_db_a 

A acbcad 

B cdebad 

C bdbcba 

D adabed 

Answer: D 

 

General knowledge 

Instructions For the following questions answer them individually 

Q.26 The Khadi and Village Industries Commission Act was passed in the year _____. 

A 1965 

B 1956 

C 1964 

D 1948 

Answer: B 

 

Q.27 Name the smallest district in India. 

A Hailakandi 

B Mahe 

C Alirajpur 

D Guntur 

Answer: B 

 

Q.28 Who was the first Indian to win the Miss Universe title? 

A Reita Faria 

B Persis Khambatta 

C Sushmita Sen 

D Madhu Sapre 

Answer: C 

 

Q.29 Name the western most point of India. 

A Ghuar Mota 

B Amreli 

C Bharuch 

D Dahod 

Answer: A 

 

Q.30 _____ is a hearing-impaired Indian golfer who won her first professional title at the age of 18. In March 2019, she became the second Indian woman golfer to win on the ‘Ladies European Tour’. 

A Diksha Dagar 

B Gursimar Badwal 

C Aditi Ashok 

D Vani Kapoor 

Answer: A 

 

Q.31 Wilson’s disease is an inherited disorder in which excessive amounts of _____ accumulate in the body. 

A sodium 

B calcium 

C iron 

D copper 

Answer: D 

 

Q.32 Name the oldest operating port in India built by the British East India Company. 

A Chennai Port 

B Kolkata Port 

C Mumbai Port 

D Vishakapatnam Port 

Answer: B

 

Q.33 An adult human body has _____ bones. 

A 208 

B 206 

C 312 

D 300 

Answer: B 

 

Q.34 _____ was named the cleanest city in the ‘Swachh Survekshan Survey 2019’. 

A Indore 

B Ujjain 

C Mysuru 

D Surat 

Answer: A 

 

Q.35 People above the age of _____ years are NOT eligible for the Pradhan Mantri Shram Yogi Mandhan Yojana. 

A 60 

B 40 

C 50 

D 45 

Answer: B 

 

Q.36 _____ is the hottest planet in the solar system. 

A Mercury 

B Earth 

C Mars 

D Venus 

Answer: D 

 

Q.37 The Pradhan Mantri Shram Yogi Mandhan Yojana ensures old age protection for unorganised workers whose monthly income is less than or equal to _____. 

A ₹10,000 

B ₹12,000 

C ₹15,000 

D ₹20,000 

Answer: C 

 

Q.38 The Ayushman Bharat Scheme introduced by the Government of India provides medical coverage of upto ₹ _____ per family per year. 

A 5 lakhs 

B 2 lakhs 

C 7 lakhs 

D 1 lakh 

Answer: A 

 

Q.39 Which team won the Ranji Trophy 2018-19? 

A Vidarbha 

B Kerala 

C Saurashtra 

D Karnataka 

Answer: A 

 

Q.40 Which of the following is NOT a part of the ‘Char Dham Yatra’ in North India? 

A Vaishno Devi 

B Badrinath 

C Kedarnath 

D Gangotri 

Answer: A 

 

Q.41 The movement of the local farmers of Bardoli in Gujarat against the British in 1928 was led by _____. 

A Lal Bahadur Shashtri 

B Vallabhbhai Patel 

C Lokmanya Tilak 

D Jawaharlal Nehru 

Answer: B 

 

Q.42 The Micro, Small and Medium Enterprises Development Act was passed in the year ____. 

A 2008 

B 2006 

C 2004 

D 2002 

Answer: B 

 

Q.43 The East India Company sent Captain Will i am Hawkins to the court of Emperor _____ in 1608 to seek permission to open a factory at Surat. 

A Jahangir 

B Humayun 

C Akbar 

D Shah Jahan 

Answer: A 

 

Q.44 What was the historical name of the city of Guwahati? 

A Purushapura 

B Pragjyotishpur 

C Machilipattnam 

D Bhagyanagara 

Answer: B 

 

Q.45 The ‘Instrument of Surrender’ which ended the Portuguese rule in India was signed on _____. 

A 19th December 1961 

B 29th December 1951 

C 15th August 1947 

D 26th January 1948 

Answer: A 

 

Q.46 As of 2018, India is the leading milk producing country in the world, accounting for approximately _____ of the global market share. 

A 22% 

B 19% 

C 15% 

D 17% 

Answer: B 

 

Q.47 In 2015, aircraft maker Boeing developed a metal called _____, which is lighter than air and stronger than steel. 

A Molybdenum 

B Titanium 

C Microlattice 

D Iridium 

Answer: C 

 

Q.48 _____ was the first Chief Election Commissioner of India. 

A M S Gill 

B T N Seshan 

C Sukumar Sen 

D Sunil Arora 

Answer: C 

 

Q.49 The approximate diameter of the sun is _____ times the diameter of Earth. 

A 92 

B 109 

C 123 

D 146 

Answer: B 

 

Q.50 During the Sao Joao festivities, Goans present _____ to each other. 

A fruits 

B beer 

C fish 

D flowers 

Answer: A 

 

Quantitative Aptitude 

Instructions For the following questions answer them individually 

Q.51 Table shows the percentage distribution of the expenditure incurred on different items for publishing a book.

Expenditure on Royalty is less than that on Printing by: 

A 10% 

B 20% 

C 25% 

D 15% 

Answer: C 

 

Q.52 A starts walking at 4 kmph and after 4 hours, B starts cycling from the same point as that of A, in the same direction at 10 kmph. After how much distance from the starting point will B catch up with A (Correct to two decimal places)? 

A 25.67 km 

B 26.67 km 

C 24.67 km 

D 23.67 km 

Answer: B 

 

Q.53 Table shows the annual Expenditure of a Company (in Lakh Rupees) over the years. 

What is the average amount of Interest on loans (in Lakh rupees) which the company paid during the period 1998 to 2002? 

A 34.18 

B 33.72 

C 32.43 

D 36.66 

Answer: D 

 

Q.54 Table shows the percentage of marks obtained by seven students in six different subjects in an examination. The numbers in the brackets are the maximum marks in each subject. 

What are the average marks obtained by all the seven students in Physics? (Correct to two decimal places) 

A 89.14 

B 91.16 

C 77.26 

D 93.14 

Answer: A 

 

Q.55 Table shows the sales of books (in thousands) from six branches of a publishing company during 2000 and 2001.

What is the total sales of books from branches B1, B3 and B6 together for both the years (in thousands)? 

A 240 

B 310 

C 650 

D 540 

Answer: D 

 

Q.56 If 85% of a number is added to 75, then the result is the number itself. The number is: 

A 500 

B 200 

C 100 

D 300 

Answer: A 

 

Q.57 The value of [ (sin224 + sin266)/(cos224 + cos266) +sin261 + cos61 sin29]is: 

A 1 

B 3 

C 2 

D 0 

Answer: C 

 

Q.58 ABCD is a cyclic quadrilateral such that AB is the diameter of the circle circumscribing it and ∠ADC = 129. Then, ∠BAC is equal to: 

A 61

B 49

C 39

D 61

Answer: C 

 

Q.59 The average of 27 numbers is zero. Out of them, how many may be greater than zero, at the most? 

A 0 

B 26 

C 20 

D 15 

Answer: B 

 

Q.60 PA and PB are tangents to a circle with centre O, from a point P outside the circle, and A and B are points on the circle. If ∠APB = 40, then ∠OAB is equal to: 

A 40

B 20

C 50

D 25

Answer: B

 

Q.61 3 men, 4 women and 6 boys together can complete a work in 6 days. A woman does triple the work a man does and a boy does half the work a man does. How many women alone will be able to complete this work in 4 days? 

A 7 

B 8 

C 6 

D 9 

Answer: D 

 

Q.62 What is the difference between a single discount of 30% and a single discount equivalent to two successive discounts of 25% and 5%, being given on shopping of ₹2,000? 

A ₹20 

B ₹25 

C No difference 

D ₹15 

Answer: B 

 

Q.63 A sphere of radius 6 cm is melted and recast into spheres of radius 2 cm each. How many such spheres can be made? 

A 25 

B 27 

C 36 

D 24 

Answer: B 

 

Q.64 If a sum amounts to ₹2,190 in four years and ₹2,409 in five years at compound interest, when the interest is compounded yearly, then the annual rate of interest is: 

A 11% 

B 10% 

C 9% 

D 8% 

Answer: B 

 

Q.65 When an integer n is divided by 8, the remainder is 3. What will be the remainder if 6n-1 is divided by 8 ? 

A 2 

B 0 

C 4 

D 1 

Answer: D 

 

Q.66 If a3b3 = 208 and a b = 4, then (a + b)2ab is equal to: 

A 52 

B 42 

C 32 

D 38 

Answer: A 

 

Q.67 If x + 1/x= 5, then x 3 + 1/x3is equal to: 

A 130 

B 110 

C 145 

D 125 

Answer: B 

 

Q.68 If 3cos2 A + 7sin2 A = 4 , then what is the value of cotA , given that A is an acute angle? 

A 1 

B 23

C 13

D 3 

Answer: D 

 

Q.69 If (x − 5)3 + (x − 6)3 + (x − 7)3 = 3(x − 5)(x − 6)(x − 7) , then what is the value of x ? 

A 5 

B 6 

C 18 

D 7 

Answer: B 

 

Q.70 If tanx = cot(45 + 2x) , then what is value of x? 

A 15∘ 

B 20∘ 

C 45∘ 

D 45∘ 

Answer: A 

 

Q.71 If the selling price of 40 articles is equal to the cost price of 50 articles, then the percentage loss or gain is:

A 20% loss 

B 25% loss 

C 20% gain 

D 25% gain 

Answer: D 

 

Q.72 In △ABC, ∠A = 50. Its sides AB and AC are produced to the point D and E. If the bisectors of the ∠CBD and ∠BCE meet at the point O, then ∠BOC will be equal to: 

A 40

B 65

C 75

D 55

Answer: B 

 

Q.73 A earns ₹180 per hour and works for 7 hours per day. B earns ₹160 per hour and works for 5 hours per day. What is the ratio of per day wages of A and B? 

A 40 : 61 

B 63 : 40 

C 20 : 30 

D 33 : 20 

Answer: B 

 

Q.74 If the radius of the circumcircle of an equilateral triangle is 8 cm, then the measure of radius of its incircle is: 

A 16cm 

B 4cm 

C 12cm 

D 8 cm 

Answer: B 

Q.75 is equal to: 

A ¹⁵/₄

B ³⁹/₈

C 4 

D 3

Answer: B 

 

English 

Instructions For the following questions answer them individually 

Q.76 Select the most appropriate antonym of the given word. ILLUMINATE 

A add 

B erase 

C light 

D darken 

Answer: D 

 

Q.77 Select the correct passive form of the given sentence. Ananya plucks fresh flowers from the garden every day. 

A Fresh flowers have been plucked by Ananya from the garden every day. 

B Fresh flowers were plucked by Ananya from the garden every day. 

C Fresh garden is plucked by Ananya from the flowers every day. 

D Fresh flowers are plucked by Ananya from the garden every day. 

Answer: D 

 

Q.78 Select the most appropriate option to fill in the blank. Colours, they say have the power to calm, pacify and relax; they can energise, activate and _______. 

A involve 

B envigorate 

C enrage 

D interest 

Answer: B 

 

Q.79 Select the word which means the same as the group of words given. An instrument for measuring the atmospheric pressure 

A altometer 

B barometer 

C ammeter 

D thermometer 

Answer: B 

 

Q.80 Select the most appropriate option to fill in the blank. In ancient Greece, women were not allowed to _______ in the Olympic Games. 

A comply 

B collide 

C compete 

D cope 

Answer: C 

 

Q.81 Select the most appropriate synonym of the given word. FUSE (Verb) 

A correct 

B break 

C unused 

D combine 

Answer: D 

 

Q.82 Select the correctly spelt word. 

A jewlry 

B scramble 

C defnition 

D acceptence 

Answer: B 

 

Q.83 Select the most appropriate synonym of the given word. ATTRIBUTE 

A quality 

B praise 

C respect 

D speech 

Answer: A 

 

Q.84 Select the most appropriate meaning of the underlined idiom in the given sentence. The dog played in the pond to his heart’s content. 

A avoiding getting too wet 

B till he was completely sick 

C as much as he wanted to 

D only upto his legs 

Answer: C 

 

Q.85 Select the most appropriate meaning of the underlined idiom in the given sentence. “These glasses suit you to a T ,” said Ria to Vandana. 

A look very good on your face 

B need to be worn with a suit 

C make you look weird 

D are not suitable for your work 

Answer: A 

 

Q.86 In the sentence identify the segment which contains the grammatical error. Entering the hall, the show had started. 

A had started 

B the hall 

C Entering 

D the show 

Answer: C 

 

Q.87 Select the most appropriate option to substitute the underlined segment in the given sentence. If no substitution is required, select No improvement. An old misunderstanding exists among the two families , so they are not friendly any more. 

A will exist among the two families 

B exists between the two families 

C has exist within the two families 

D No improvement 

Answer: B 

 

Instructions In the following passage some words have been deleted. Fill in the blanks with the help of the alternatives given. Select the most appropriate option for each blank. 

Comprehension: Dusk sets in at the Varagaliar elephant camp deep within the Anamalai Tiger Reserve (ATR). It’s dinner time for the 25- year-old tusker and he’s (1)______ balls made of steamed ragi flour mixed (2)______ some rice, green gram, salt and jaggery. (3)______ are half-a-dozen Forest Department (4)______ around, including V. Ganesan, the Field Director (5)______ ATR. 

Q.88 Select the most appropriate option to fill in blank No.(1) 

A feeding 

B feeds 

C fed 

D feed 

Answer: C 

 

Q.89 Select the most appropriate option to fill in blank No.(2) 

A by 

B of 

C with 

D from 

Answer: C 

 

Q.90 Select the most appropriate option to fill in blank No.(3) 

A Here 

B There 

C Everywhere 

D Nowhere 

Answer: B 

 

Q.91 Select the most appropriate option to fill in blank No.(4) 

A employee 

B officials 

C person 

D ministers 

Answer: B 

 

Q.92 Select the most appropriate option to fill in blank No.(5) 

A on 

B between 

C among 

D of 

Answer: D 

 

Instructions For the following questions answer them individually 

Q.93 Given below are four jumbled sentences. Select the option that gives their correct order. 

A. Once the formalities are done, you can start using your account and save time and money. B. Opening a bank account can seem intimidating. 

C. Getting your account opened is just a matter of providing certain details and funding your account. 

D. Fortunately, most banks follow a standardized process. 

A CBDA 

B ADBC 

C BDCA 

D BADC 

Answer: C 

 

Q.94 In the sentence identify the segment which contains the grammatical error. “I done a lot of work today,” she said. 

A a lot of 

B work today 

C she said 

D I done 

Answer: D

 

Q.95 Select the correctly spelt word. 

A choose 

B chooze 

C chooj 

D chuse 

Answer: A 

 

Q.96 Given below are four jumbled sentences. Select the option that gives their correct order. 

A. A man on a bike had to ride on the pavement. 

B. When he tried to plunge onto the road, a car hit him. 

C. As the bus inched through the evening life, the traffic grew. 

D. There was no space on the road any more. 

A CDAB 

B CBAD 

C ADBC 

D DBCA 

Answer: A 

 

Q.97 Select the word which means the same as the group of words given. The part of a country’s government responsible for its legal system 

A judiciary 

B legislature 

C executive 

D police 

Answer: A 

 

Q.98 Select the most appropriate option to substitute the underlined segment in the given sentence. If no substitution is required, select No improvement. Scarcely had he gone out than a client came to meet him. 

A he had go out when 

B he had gone out than 

C No improvement 

D had he gone out when 

Answer: D 

 

Q.99 Select the correct active form of the given sentence. Dinesh was looked after by his grandmother when his parents went abroad. 

A Dinesh’s grandmother looks after him when his parents went abroad. 

B Dinesh’s grandmother was looking after him when his parents went abroad. 

C Dinesh’s grandmother looked after him when his parents went abroad. 

D Dinesh looked after his grandmother when his parents went abroad. 

Answer: C 

 

Q.100 Select the most appropriate antonym of the given word. DEFICIT 

A surplus 

B adhere 

C remove 

D credit 

Answer: A 

×

Hello!

Click one of our representatives below to chat on WhatsApp or send us an email to info@vidhyarthidarpan.com

×